You are on page 1of 120

A 76 year old woman with a body weight of 50 kg is undergoing an excision of a

lipoma from her forehead. It is the first time the senior house officer has performed
the procedure. He administers 30ml of 2% lignocaine to the area. The procedure is
complicated by bleeding and the patient experiences discomfort, a further 10ml of the
same anaesthetic formulation is then administered. Over the following 5 minutes the
patient complains of tinnitus and becomes drowsy. Which of the drugs listed below
should be administered?

Temazepam A.

Lorazepam B.

Naloxone C.

Intralipid 20% D.

Sodium bicarbonate 20% E.


Local anaesthetic toxicity
treatment = Intralipid

Intralipid is indicated for the treatment of local anaesthetic toxicity. In this case the
safe dose of local anaesthetic has been exceeded and is thus this lady's symptoms are
likely to represent toxicity.

Local anaesthetic toxicity

Toxicity results from either accidental intravascular injection (rapid onset of


symptoms-usually correct dose), or from excessive dosage (slower onset). Local
anaesthetic agents not only exert a membrane stabilising effect on peripheral nerves
but will also act on excitable membranes within the CNS and Heart. The inhibitory
neurones in the CNS are suppressed before the central ones. As a result the early
symptoms will typically be those of circumoral paraesthesia and tinnitus, followed by
falling GCS and eventually coma.

Management of toxicity

 Stop injecting the anaesthetic agent


 High flow 100% oxygen via face mask
 Cardiovascular monitoring
 Administer lipid emulsion (Intralipid 20%) at 1.5ml/Kg over 1 minute as a
bolus
 Consider lipid emulsion infusion, at 0.25ml/ Kg/ minute

Safe doses
10ml of lignocaine 1% contains 100mg of drug, this would constitute 70% of the
maximum safe dose in a 50 kg patient. Up to 7mg / kg can be administered if
adrenaline is added to the solution.
Doses of local anaesthetics
Dose with adrenaline Dose plain Agent
7mg/Kg 3mg/Kg Lignocaine
2mg/Kg 2mg/Kg Bupivicane
9mg/Kg 6mg/Kg Prilocaine
These are a guide only as actual doses depend on site of administration, tissue
vascularity and co-morbidities.

A patient is brought to the emergency department following a motor vehicle accident.


He is unconscious and has a deep scalp laceration. His heart rate is 120/min, blood
pressure is 80/40 mmHg, and respiratory rate is 35/min. Despite rapid administration
of 2 litres of Hartmans solution, the patient's vital signs do not change significantly.
The injury likely to explain this patient's hypotension is:

Epidural haematoma A.

Sub dural haematoma B.

Intra parenchymal brain haemorrhage C.

Base of skull fracture D.

None of the above E.


Raised intracranial hypertension: Hypertension, Bradycardia, Respiratory depression

In the patient described, hypotension and tachycardia should not be uncritically


attributed to the head injury, since these findings in the setting of blunt trauma are
suggestive of serious thoracic, abdominal, or pelvic hemorrhage. When cardiovascular
collapse occurs as a result of rising intracranial pressure, it is generally accompanied
by hypertension, bradycardia, and respiratory depression.

Head injury

 Patients who suffer head injuries should be managed according to ATLS


principles and extra cranial injuries should be managed alongside cranial
trauma. Inadequate cardiac output will compromise CNS perfusion
irrespective of the nature of the cranial injury.

Types of traumatic brain injury


Bleeding into the space between the dura mater and the skull. Often Extradural
results from acceleration-deceleration trauma or a blow to the side haematoma
of the head. The majority of extradural haematomas occur in the
temporal region where skull fractures cause a rupture of the middle
meningeal artery.
Features

 Raised intracranial pressure


 Some patients may exhibit a lucid interval

Bleeding into the outermost meningeal layer. Most commonly occur Subdural
around the frontal and parietal lobes. May be either acute or haematoma
chronic.

Risk factors include old age and alcoholism.

Slower onset of symptoms than a extradural haematoma.


Usually occurs spontaneously in the context of a ruptured cerebral Subarachnoid
aneurysm but may be seen in association with other injuries when a haemorrhage
patient has sustained a traumatic brain injury

Pathophysiology

 Primary brain injury may be focal (contusion/ haematoma) or diffuse (diffuse


axonal injury)
 Diffuse axonal injury occurs as a result of mechanical shearing following
deceleration, causing disruption and tearing of axons
 Intra-cranial haematomas can be extradural, subdural or intracerebral, while
contusions may occur adjacent to (coup) or contralateral (contre-coup) to the
side of impact
 Secondary brain injury occurs when cerebral oedema, ischaemia, infection,
tonsillar or tentorial herniation exacerbates the original injury. The normal
cerebral auto regulatory processes are disrupted following trauma rendering
the brain more susceptible to blood flow changes and hypoxia
 The Cushings reflex (hypertension and bradycardia) often occurs late and is
usually a pre terminal event

Management

 Where there is life threatening rising ICP such as in extra dural haematoma
and whilst theatre is prepared or transfer arranged use of IV mannitol/
frusemide may be required.
 Diffuse cerebral oedema may require decompressive craniotomy
 Exploratory Burr Holes have little management in modern practice except
where scanning may be unavailable and to thus facilitate creation of formal
craniotomy flap
 Depressed skull fractures that are open require formal surgical reduction and
debridement, closed injuries may be managed non operatively if there is
minimal displacement.
 ICP monitoring is appropriate in those who have GCS 3-8 and normal CT
scan.
 ICP monitoring is mandatory in those who have GCS 3-8 and Abnormal CT
scan.
 Hyponatraemia is most likely to be due to syndrome of inappropriate ADH
secretion.
 Minimum of cerebral perfusion pressure of 70mmHg in adults.
 Minimum cerebral perfusion pressure of between 40 and 70 mmHg in
children.

Interpretation of pupillary findings in head injuries


Interpretation Light response Pupil size
3rd nerve compression secondary to Sluggish or fixed Unilaterally dilated
tentorial herniation
 Poor CNS perfusion Sluggish or fixed Bilaterally dilated
 Bilateral 3rd nerve palsy

Optic nerve injury Cross reactive Unilaterally dilated


(Marcus - Gunn) or equal
 Opiates May be difficult to Bilaterally
 Pontine lesions assess constricted
 Metabolic encephalopathy

Sympathetic pathway disruption Preserved Unilaterally


constricted
A 42 year old man is admitted to surgery with acute appendicitis. He is known to have
hypertension, psoriatic arthropathy and polymyalgia rheumatica. His medical therapy
includes:
Paracetamol 1g qds
Codeine phosphate 30mg qds
Bendrofluazide 2.5 mg od
Ramipril 10mg od
Methotrexate 7.5mg once a week
Prednisolone 5mg od
You are called by the Senior House Officer to assess this man as he has become
delirious and hypotensive 2h after surgery. His blood results reveal:

132 mmol/l Na+


5.2 mmol/l K+
10 mmol/l Urea
111 µmol/l Creatinine
3.5 Glucose
158 CRP

10.2 g/dl Hb
156 * 109/l Platelets
14 * 109/l WBC
What is the most likely diagnosis?

Septic shock secondary to appendicitis A.

Neutropenic sepsis B.

Phaeochromocytoma C.

Perforated bowel D.

Addisonian crisis E.
Features of an addisonian
crisis:

 Hyponatraemia
 Hyperkalaemia
 Hypoglycaemia

This man is on steroids for polymyalgia rheumatica. Surgery can precipitate acute
adrenal deficiency. The diagnosis is further confirmed by the blood results of
hyponatraemia, hyperkalaemia and hypoglycaemia. This patient urgently needs
Hydrocortisone.

Addisonian crisis

Causes

 Sepsis or surgery causing an acute exacerbation of chronic insufficiency


(Addison's, Hypopituitarism)
 Adrenal haemorrhage eg Waterhouse-Friderichsen syndrome (fulminant
meningococcemia)
 Steroid withdrawal

Management

 Hydrocortisone 100 mg im or iv
 1 litre normal saline infused over 30-60 mins or with dextrose if
hypoglycaemic
 Continue hydrocortisone 6 hourly until the patient is stable. No
fludrocortisone is required because high cortisol exerts weak
mineralocorticoid action
 Oral replacement may begin after 24 hours and be reduced to maintenance
over 3-4 days
A 32 year old man is involved in a motorcycle accident and sustains a closed unstable
spiral tibial fracture. This is managed with an intramedullary nail. On return to the
ward he is noted to have increasing pain in the limb and on examination the limb is
swollen and tender with pain on passive stretching of the toes. The most likely
diagnosis is:

Tibial nerve neuropraxia A.

Displaced tibial nail B.

Compartment syndrome C.

Deep vein thrombosis D.

Sciatic nerve injury E.

Theme from 2009 Exam


Theme from September 2012 Exam
Severe pain in a limb should raise suspicions of compartment syndrome especially in
tibial fractures following fixation with intra medullary devices.

Compartment syndrome

 This is a particular complication that may occur following fractures (or


following ischaemia reperfusion injury in vascular patients). It is characterised
by raised pressure within a closed anatomical space.
 The raised pressure within the compartment will eventually compromise tissue
perfusion resulting in necrosis. The two main fractures carrying this
complication include supracondylar fractures and tibial shaft injuries.

Symptoms and signs

 Pain, especially on movement (even passive)


 Parasthesiae
 Pallor may be present
 Arterial pulsation may still be felt as the necrosis occurs as a result of
microvascular compromise
 Paralysis of the muscle group may occur

Diagnosis

 Is made by measurement of intracompartmental pressure measurements.


Pressures in excess of 20mmHg are abnormal and >40mmHg is diagnostic.

Treatment
 This is essentially prompt and extensive fasciotomies
 In the lower limb the deep muscles may be inadequately decompressed by the
inexperienced operator when smaller incisions are performed
 Myoglobinuria may occur following fasciotomy and result in renal failure and
for this reason these patients require aggressive IV fluids
 Where muscle groups are frankly necrotic at fasciotomy they should be
debrided and amputation may have to be considered
 Death of muscle groups may occur within 4-6 hours

A 28 year old man is involved in a road traffic accident and sustains a flail chest
injury. On arrival in the emergency department he is hypotensive. On examination he
has an elevated jugular venous pulse and auscultation of the heart reveals quiet heard
sounds. What is the most likely diagnosis?

Pneumothorax A.

Myocardial contusion B.

Cardiac tamponade C.

Haemothorax D.

Ventricular septal defect E.

Theme from 2010 exam


The presence of a cardiac tamponade is suggested by Becks Triad:

 Hypotension
 Muffled heart sounds
 Raised JVP

Thoracic trauma

Types of thoracic trauma

 Often laceration to lung parenchyma with flap Tension


 Pressure develops in thorax pneumothorax
 Most common cause is mechanical ventilation in patient
with pleural injury
 Symptoms overlap with cardiac tamponade, hyper-
resonant percussion note is more likely in tension
pnemothorax

 Chest wall disconnects from thoracic cage Flail chest


 Multiple rib fractures (at least two fractures per rib in at
least two ribs)
 Associated with pulmonary contusion
 Abnormal chest motion
 Avoid over hydration and fluid overload

 Most common cause is lung laceration with air leakage Pneumothorax


 Most traumatic pneumothoraces should have a chest
drain
 Patients with traumatic pneumothorax should never be
mechanically ventilated until a chest drain is inserted

 Most commonly due to laceration of lung, intercostal Haemothorax


vessel or internal mammary artery
 Haemothoraces large enough to appear on CXR are
treated with large bore chest drain
 Surgical exploration is warranted if >1500ml blood
drained immediately

 Beck's triad: elevated venous pressure, reduced arterial Cardiac tamponade


pressure, reduced heart heart sounds.
 Pulsus paradoxus
 May occur with as little as 100ml blood

 Most common potentially lethal chest injury Pulmonary


 Arterial blood gases and pulse oximetry important contusion
 Early intubation within an hour if significant hypoxia

 Usually occurs secondary to chest wall injury Blunt cardiac injury


 ECG may show features of myocardial infarction
 Sequelae: hypotension, arrhythmias, cardiac wall motion
abnormalities

 Deceleration injuries Aorta disruption


 Contained haematoma
 Widened mediastinum

 Most due to motor vehicle accidents and blunt trauma Diaphragm


causing large radial tears (laceration injuries result in disruption
small tears)
 More common on left side
 Insert gastric tube, which will pass into the thoracic
cavity

 Entrance wound in one haemothorax and exit Mediastinal


wound/foreign body in opposite haemothorax traversing wounds
 Mediastinal haematoma or pleural cap suggests great
vessel injury
 Mortality is 20%
A 52 year old male type 2 diabetic is admitted to the vascular ward for a femoral
popliteal bypass. He suddenly develops expressive dysphasia and marked right sided
weakness. The Senior house officer arranges a CT head scan which shows a 60% left
middle cerebral artery territory infarct. There are no beds on the stroke unit.
Overnight the patient becomes unresponsive and a CT head confirms no bleed. What
is the next best management option?

IV heparin A.

Clopidogrel B.

Burr hole surgery C.

Aspirin D.

Hemicraniectomy E.

The likely cause for the reduced consciousness is raised intracranial pressure due to
increasing cerebral oedema related to the infarct. In this situation, urgent
neurosurgical review is needed for possible decompressive hemicraniectomy to
relieve the pressure. Ideally no further antiplatelet or anticoagulation therapy should
be given until a plan for surgery is confirmed.

Indications for hemicraniectomy include:

 Age under 60 years


 Clinical deficit in middle cerebral artery territory
 Decreased consciousness
 >50% territory infarct

Stroke: types

 Presents with headache, vomiting, loss of Primary intracerebral


consciousness haemorrhage (PICH, c.
10%)
 Involves middle and anterior cerebral arteries Total anterior circulation
 Hemiparesis/hemisensory loss infarcts (TACI, c. 15%)
 Homonymous hemianopia
 Higher cognitive dysfunction e.g. Dysphasia

 Involves smaller arteries of anterior circulation Partial anterior circulation


e.g. upper or lower division of middle cerebral infarcts (PACI, c. 25%)
artery
 Higher cognitive dysfunction or two of the three
TACI features
 Involves perforating arteries around the internal Lacunar infarcts (LACI, c.
capsule, thalamus and basal ganglia 25%)
 Present with either isolated hemiparesis,
hemisensory loss or hemiparesis with limb ataxia

 Vertebrobasilar arteries Posterior circulation


 Presents with features of brainstem damage infarcts (POCI, c. 25%)
 Ataxia, disorders of gaze and vision, cranial
nerve lesions

 Wallenberg's syndrome Lateral medullary


 Ipsilateral: ataxia, nystagmus, dysphagia, facial syndrome (posterior
numbness, cranial nerve palsy e.g. Horner's inferior cerebellar artery)
 Contralateral: limb sensory loss

 Ipsilateral III palsy Weber's syndrome


 Contralateral weakness

--------------------------------------

Anterior cerebral artery

 Contralateral hemiparesis and sensory loss, lower extremity > upper


 Disconnection syndrome

Middle cerebral artery

 Contralateral hemiparesis and sensory loss, upper extremity > lower


 Contralateral hemianopia
 Aphasia (Wernicke's)
 Gaze abnormalities

Posterior cerebral artery

 Contralateral hemianopia with macular sparing


 Disconnection syndrome

Lacunar

 Present with either isolated hemiparesis, hemisensory loss or hemiparesis with


limb ataxia

Lateral medulla (posterior inferior cerebellar artery)


 Ipsilateral: ataxia, nystagmus, dysphagia, facial numbness, cranial nerve palsy
e.g.

Horner's

 Contralateral: limb sensory loss

Pontine

 VI nerve: horizontal gaze palsy


 VII nerve
 Contralateral hemiparesis

Theme: Trauma

Tension pneumothorax A.
Haemopericardium B.
Haemothorax C.
Aortic transection D.
Ruptured spleen E.
Duodeno-jejunal flexure disruption F.
Aorto iliac disruption G.
Ileo-colic junction disruption H.

For each scenario please select the most likely injury. Each option may be used once,
more than once or not at all.

A 24 year old motorist is involved in a road traffic accident in which he collides 7.


with the wall of a tunnel in a head on car crash, speed 85mph. He is wearing a
seatbelt and the airbags have deployed. When rescuers arrive he is lucid and
conscious and then dies suddenly.

Aortic transection

Aortic transections typically occur distal to the ligamentum arteriosum. A


temporary haematoma may prevent the immediate death that usually occurs.
This is a deceleration injury. A widened mediastinum may be seen on x-ray.

A 30 year old women is involved in a road traffic accident she is a passenger in 8.


a car involved in a head on collision with another vehicle. Her car is travelling
at 60mph. She has been haemodynamically stable throughout with only minimal
tachycardia. On examination she has marked abdominal tenderness and a large
amount of intra abdominal fluid on CT scan
You answered Aorto iliac disruption

The correct answer is Duodeno-jejunal flexure disruption

This is another site of sudden deceleration injury. Given the large amount of
free fluid, if it were blood, then a greater degree of haemodynamic instability
would be expected.

A 17 year old boy is involved in a motorcycle accident in which he is thrown 9.


from his motorcycle. On admission he has distended neck veins and a weak
pulse. The trachea is central.

Haemopericardium

This is most likely a cardiac tamponade produced by haemopericardium. As


little as 100ml of blood may result in tamponade as the pericardial sac is not
distensible. Diagnosis is suggested by muffled heart sounds, paradoxical pulse
and jugular vein distension.

Thoracic trauma

Types of thoracic trauma

 Often laceration to lung parenchyma with flap Tension


 Pressure develops in thorax pneumothorax
 Most common cause is mechanical ventilation in patient
with pleural injury
 Symptoms overlap with cardiac tamponade, hyper-
resonant percussion note is more likely in tension
pnemothorax

 Chest wall disconnects from thoracic cage Flail chest


 Multiple rib fractures (at least two fractures per rib in at
least two ribs)
 Associated with pulmonary contusion
 Abnormal chest motion
 Avoid over hydration and fluid overload

 Most common cause is lung laceration with air leakage Pneumothorax


 Most traumatic pneumothoraces should have a chest
drain
 Patients with traumatic pneumothorax should never be
mechanically ventilated until a chest drain is inserted
 Most commonly due to laceration of lung, intercostal Haemothorax
vessel or internal mammary artery
 Haemothoraces large enough to appear on CXR are
treated with large bore chest drain
 Surgical exploration is warranted if >1500ml blood
drained immediately

 Beck's triad: elevated venous pressure, reduced arterial Cardiac tamponade


pressure, reduced heart heart sounds.
 Pulsus paradoxus
 May occur with as little as 100ml blood

 Most common potentially lethal chest injury Pulmonary


 Arterial blood gases and pulse oximetry important contusion
 Early intubation within an hour if significant hypoxia

 Usually occurs secondary to chest wall injury Blunt cardiac injury


 ECG may show features of myocardial infarction
 Sequelae: hypotension, arrhythmias, cardiac wall motion
abnormalities

 Deceleration injuries Aorta disruption


 Contained haematoma
 Widened mediastinum

 Most due to motor vehicle accidents and blunt trauma Diaphragm


causing large radial tears (laceration injuries result in disruption
small tears)
 More common on left side
 Insert gastric tube, which will pass into the thoracic
cavity

 Entrance wound in one haemothorax and exit Mediastinal


wound/foreign body in opposite haemothorax traversing wounds
 Mediastinal haematoma or pleural cap suggests great
vessel injury
 Mortality is 20%

A 14-year-old boy is admitted to the acute surgical unit with appendicitis. He is


normally fit and well. Apart from metoclopramide, the patient has had no other
medications. The nursing staff contact you as the patient is acting strange. On
examination he is agitated, has a clenched jaw and his eyes are deviated upwards.
What is the most likely diagnosis?

Functional disorder A.

Malignant hyperthermia B.
Oculogyric crisis C.

Epilepsy D.

Serotonin syndrome E.

This is a classic description of an oculogyric crisis, a form of extrapyramidal disorder.


An oculogyric crisis is an acute dystonic reaction. This is precipitated by
antipsychotics (haloperidol) and metoclopramide in susceptible individuals with a
genetic predisposition to this. Treatment is with procyclidine IM.

Oculogyric crisis

An oculogyric crisis is a dystonic reaction to certain drugs or medical conditions

Features

 Restlessness, agitation
 Involuntary upward deviation of the eyes

Causes

 Phenothiazines
 Haloperidol
 Metoclopramide
 Postencephalitic Parkinson's disease

Management

 Procyclidine

A 6 year old boy pulls over a kettle and suffers superficial partial thickness burns to
his legs. Which of the following will not occur?

Preservation of hair follicles A.

Formation of vesicles or bullae B.

Damage to sweat glands C.

Healing by re-epithelialisation D.

The burn area will be painful E.

Partial thickness burns are divided into superficial and deep burns, however, this is
often not possible on initial assessment and it may be a week or more before the
distinction is clear cut. Dermal appendages are, by definition, intact. Superficial
partial thickness burns will typically heal by re-epithelialisation, deeper burns will
heal with scarring.

Burns

Types of burn

Management Blanching Skin Skin layers Type of burn


appearance affected
Yes Red, moist Epidermis Epidermal/Superficial
Normally heals Yes Pale, dry Epidermis and Superficial partial
with no part of papillary thickness
intervention dermis affected
Needs surgical No Mottled red Epidermis, whole Deep partial thickness
intervention colour papillary dermis
(depending on affected
site)
Burns centre No Dry, leathery Whole skin layer Full thickness
hard wound and subcutaneous
tissue affected

Depth of burn assessment

 Bleeding on needle prick


 Sensation
 Appearance
 Blanching to pressure

Percentage burn estimation


Lund Browder chart: most accurate even in children
Wallace rule of nines
Palmar surface: surface area palm = 0.8% burn

>15% body surface area burns in adults needs urgent burn fluid resuscitation

Transfer to burn centre if:

 Need burn shock resuscitation


 Face/hands/genitals affected
 Deep partial thickness or full thickness burns
 Significant electrical/chemical burns

Escharotomies

 Indicated in circumferential full thickness burns to the torso or limbs.


 Careful division of the encasing band of burn tissue will potentially improve
ventilation (if the burn involves the torso), or relieve compartment syndrome
and oedema (where a limb is involved)

References
www.euroburn.org/e107files/downloads/guidelinesburncare.pdf

Hettiaratchy S & Papini R. Initial management of a major burn: assessment and


resuscitation. BMJ 2004;329:101-103
You are called to the acute surgical unit. A patient who has short gut syndrome has
developed a broad complex tachycardia. You suspect a diagnosis of ventricular
tachycardia. What is the most likely precipitant?

Hypoglycaemia A.

Bisoprolol B.

Hypomagnesaemia C.

Dehydration D.

Hyperthyroidism E.

Ventricular tachcardia

Ventricular tachycardia (VT)is broad-complex tachycardia originating from a


ventricular ectopic focus. It has the potential to precipitate ventricular fibrillation and
hence requires urgent treatment.

There are two main types of VT:

 monomorphic VT: most commonly caused by myocardial infarction


 polymorphic VT: A subtype of polymorphic VT is torsades de pointes which
is precipitated by prolongation of the QT interval. The causes of a long QT
interval are listed below

Causes of a prolonged QT interval

Other Drugs Congenital

 electrolyte:  amiodarone, sotalol,  Jervell-Lange-


hypocalcaemia, class 1a Nielsen syndrome
hypokalaemia, antiarrhythmic (includes deafness
hypomagnesaemia drugs and is due to an
 acute myocardial  tricyclic abnormal potassium
infarction antidepressants,
 myocarditis fluoxetine channel)
 hypothermia  chloroquine  Romano-Ward
 subarachnoid  terfenadine* syndrome (no
haemorrhage  erythromycin deafness)

Based on the current guidelines, which option regarding management of head injuries
is false?

Opiates should be avoided A.

Consider intubation if the GCS is <8 or = 8 B.

Immediate CT head if there is > 1 episode of vomiting C.

Half hourly GCS assessment until GCS is 15 D.

Contact neurosurgeons if suspected penetrating injury E.

Pain should be controlled, with opiates preferably, as this avoids distress and
hypertension post injury.

Head injury management- NICE Guidelines

Summary of guidelines

 All patients should be assessed within 15 minutes on arrival to A&E


 Document all 3 components of the GCS
 If GCS <8 or = to 8, consider stabilising the airway
 Treat pain with low dose IV opiates (if safe)
 Full spine immobilisation until assessment if:

- GCS < 15
- neck pain/tenderness
- paraesthesia extremities
- focal neurological deficit
- suspected c-spine injury

If a c-spine injury is suspected a 3 view c-spine x-ray is indicated. CT c-spine is


preferred if:
- Intubated
- GCS <13
- Normal x-ray but continued concerns regarding c-spine injury

Immediate CT head (within 1h) if:

 GCS < 13 on admission


 GCS < 15 2h after admission
 Suspected open or depressed skull fracture
 Suspected skull base fracture (panda eyes, Battle's sign, CSF from nose/ear,
bleeding ear)
 Focal neurology
 Vomiting > 1 episode
 Post traumatic seizure
 Coagulopathy

Contact neurosurgeon if:

 Persistent GCS < 8 or = 8


 Unexplained confusion > 4h
 Reduced GCS after admission
 Progressive neurological signs
 Incomplete recovery post seizure
 Penetrating injury
 Cerebrospinal leak

Observations

 1/2 hourly GCS until 15

Reference
http://guidance.nice.org.uk/CG56/QuickRefGuide/pdf/English
22 year old man suffers 20% partial and full thickness burns in a house fire. There is
an associated inhalational injury. It is decided to administer intravenous fluids to
replace fluid losses. Which of the following intravenous fluids should be used for
initial resuscitation?

Dextran 40 A.

5% Dextrose B.

Fresh frozen plasma C.

Hartmans solution D.

Blood E.

In most units a crystalloid such as Hartmans (Ringers lactate) is administered initially.


Controversy does remain and some units do prefer colloid. Should this leak in the
interstial tissues this may increase the risk of oedema.

Fluid resuscitation burns


Indication: >15% total body area burns in adults (>10% children)

 The main aim of resuscitation is to prevent the burn deepening


 Most fluid is lost 24h after injury
 First 8-12h fluid shifts from intravascular to interstitial fluid compartments
 Therefore circulatory volume can be compromised. However fluid
resuscitation causes more fluid into the interstitial compartment especially
colloid (therefore avoided in first 8-24h)
 Protein loss occurs

Fluid resuscitation formula


Parkland formula
(Crystalloid only e.g. Hartman's solution/Ringers' lactate)
Total fluid requirement in 24 hours =
4 ml x (total burn surface area (%)) x (body weight (kg))

 50% given in first 8 hours


 50% given in next 16 hours

Resuscitation endpoint:Urine output of 0.5-1.0 ml/kg/hour in adults (increase rate of


fluid to achieve this)
Points to note:

 Starting point of resuscitation is time of injury


 Deduct fluids already given

After 24 hours

 Colloid infusion is begun at a rate of 0.5 ml x(total burn surface area


(%))x(body weight (kg))
 Maintenance crystalloid (usually dextrose-saline) is continued at a rate of 1.5
ml x(burn area)x(body weight)
 Colloids used include albumin and FFP
 Antioxidants, such as vitamin C, can be used to minimize oxidant-mediated
contributions to the inflammatory cascade in burns
 High tension electrical injuries and inhalation injuries require more fluid
 Monitor: packed cell volume, plasma sodium, base excess, and lactate

A 23 year old man sustains a severe facial fracture and reconstruction is planned.
Which of the following investigations will facilitate pre-operative planning?

Mandibular tomography A.

Magnetic resonance scan of face B.


Skull X-ray C.

Computerised tomography of the head D.

Orthopantomogram E.

Theme from 2011 Exam

Significant facial fractures may have intracranial communication. CT scanning will


allow delineation of injury extent and 3D reconstruction images can be created. An
Orthopantomogram (OPT) will provide good images of mandible and surrounding
bony structures but will not give intracranial detail. A skull x-ray lacks the detail for
modern practice.

Craniomaxillofacial injuries

Craniomaxillofacial injuries in the UK are due to:

 Interpersonal violence (52%)


 Motor vehicle accidents (16%)
 Sporting injuries (19%)
 Falls (11%)

Le Fort Fractures
Feature Grade
The fracture extends from the nasal septum to the lateral pyriform rims, travels Le Fort
horizontally above the teeth apices, crosses below the zygomaticomaxillary 1
junction, and traverses the pterygomaxillary junction to interrupt the pterygoid
plates.
These fractures have a pyramidal shape and extend from the nasal bridge at or Le Fort
below the nasofrontal suture through the frontal process of the maxilla, 2
inferolaterally through the lacrimal bones and inferior orbital floor and rim
through or near the inferior orbital foramen, and inferiorly through the anterior
wall of the maxillary sinus; it then travels under the zygoma, across the
pterygomaxillary fissure, and through the pterygoid plates.
These fractures start at the nasofrontal and frontomaxillary sutures and extend Le Fort
posteriorly along the medial wall of the orbit through the nasolacrimal groove 3
and ethmoid bones. The thicker sphenoid bone posteriorly usually prevents
continuation of the fracture into the optic canal. Instead, the fracture continues
along the floor of the orbit along the inferior orbital fissure and continues
superolaterally through the lateral orbital wall, through the zygomaticofrontal
junction and the zygomatic arch. Intranasally, a branch of the fracture extends
through the base of the perpendicular plate of the ethmoid, through the vomer,
and through the interface of the pterygoid plates to the base of the sphenoid.
This type of fracture predisposes the patient to CSF rhinorrhea more
commonly than the other types.
Ocular injuries
Superior orbital fissure syndrome
Severe force to the lateral wall of the orbit resulting in compression of neurovascular
structures. Results in :

 Complete opthalmoplegia and ptosis (Cranial nerves 3, 4, 6 and nerve to


levator palpebrae superioris)
 Relative afferent pupillary defect
 Dilatation of the pupil and loss of accommodation and corneal reflexes
 Altered sensation from forehead to vertex (frontal branch of trigeminal nerve)

Orbital blow out fracture


Typically occurs when an object of slightly larger diameter than the orbital rim strikes
the incompressible eyeball. The bone fragment is displaced downwards into the antral
cavity, remaining attached to the orbital periosteum. Periorbital fat may be herniated
through the defect, interfering with the inferior rectus and inferior oblique muscles
which are contained within the same fascial sheath. This prevents upward movement
and outward rotation of the eye and the patient experiences diplopia on upward gaze.
The initial bruising and swelling may make assessment difficult and patients should
usually be reviewed 5 days later. Residual defects may require orbital floor
reconstruction.

Nasal Fractures

 Common injury
 Ensure new and not old deformity
 Control epistaxis
 CSF rhinorrhoea implies that the cribriform plate has been breached and
antibiotics will be required.
 Usually best to allow bruising and swelling to settle and then review patient
clinically. Major persistent deformity requires fracture manipulation, best
performed within 10 days of injury.

Retrobulbar haemorrhage
Rare but important ocular emergency. Presents with:

 Pain (usually sharp and within the globe)


 Proptosis
 Pupil reactions are lost
 Paralysis (eye movements lost)
 Visual acuity is lost (colour vision is lost first)

May be the result of Le Fort type facial fractures.

Management:
 Mannitol 1g/Kg as 20% infusion, Osmotic diuretic, Contra-indicated in
congestive heart failure and pulmonary oedema
 Acetazolamide 500mg IV, (Monitor FBC/U+E) Reduces aqueous pressure by
inhibition of carbonic anhydrase (used in glaucoma)
 Dexamethasone 8mg orally or intravenously
 In a traumatic setting an urgent catholysis may be needed prior to definitive
surgery.

Consider
Papaverine 40mg smooth muscle relaxant
Dextran 40 500mls IV improves perfusion
Theme: Treatment of burns

3500ml Human albumin solution A.


3300ml Hartmans solution B.
4500ml Hartmans solution C.
6600ml Hartmans solution D.
5500ml Hartmans solution E.
5500ml Human albumin solution F.
6000ml Human albumin solution G.
Fluid resuscitation not required H.
5500ml Fresh frozen plasma I.

For the burns patients described below please determine the correct volume and type
of intravenous fluid (if required) for the first 8 hours of care. Each option may be used
once, more than once or not at all.

A 25 year old male is admitted having been involved in a house fire. His weight 1.
on admission is 75 kg and he has suffered 22% full thickness burns to his torso
and limbs.

3300ml Hartmans solution

Theme from April 2012 Exam


Using the Parkland formula calculated below:
4 x 22 x 75/2= 3300ml.
Remember half the total is administered in the first 8 hours.
Hartmans is the preferred agent.

A 25 year old male is admitted having been involved in a house fire. His weight 2.
on admission is 85 kg and he has suffered 19% full thickness burns to his torso
and limbs.

3300ml Hartmans solution


Using the Parkland formula calculated below:
4 x 19 x 85/2= 3230ml
Therefore 3300ml is the nearest approximate amount.

A 25 year old male is admitted having been involved in a house fire. His weight 3.
on admission is 65 kg and he has suffered 25% full thickness burns to his torso
and limbs.

3300ml Hartmans solution

Using the Parkland formula calculated below:


4 x 25 x 65/2= 3250ml
Therefore 3300ml is the nearest approximate amount.

Fluid resuscitation burns

Indication: >15% total body area burns in adults (>10% children)

 The main aim of resuscitation is to prevent the burn deepening


 Most fluid is lost 24h after injury
 First 8-12h fluid shifts from intravascular to interstitial fluid compartments
 Therefore circulatory volume can be compromised. However fluid
resuscitation causes more fluid into the interstitial compartment especially
colloid (therefore avoided in first 8-24h)
 Protein loss occurs

Fluid resuscitation formula


Parkland formula
(Crystalloid only e.g. Hartman's solution/Ringers' lactate)
Total fluid requirement in 24 hours =
4 ml x (total burn surface area (%)) x (body weight (kg))

 50% given in first 8 hours


 50% given in next 16 hours

Resuscitation endpoint:Urine output of 0.5-1.0 ml/kg/hour in adults (increase rate of


fluid to achieve this)
Points to note:

 Starting point of resuscitation is time of injury


 Deduct fluids already given
After 24 hours

 Colloid infusion is begun at a rate of 0.5 ml x(total burn surface area


(%))x(body weight (kg))
 Maintenance crystalloid (usually dextrose-saline) is continued at a rate of 1.5
ml x(burn area)x(body weight)
 Colloids used include albumin and FFP
 Antioxidants, such as vitamin C, can be used to minimize oxidant-mediated
contributions to the inflammatory cascade in burns
 High tension electrical injuries and inhalation injuries require more fluid
 Monitor: packed cell volume, plasma sodium, base excess, and lactate

A 23 year old man is stabbed in the right upper quadrant and is haemodynamically
unstable. A laparotomy is performed and the liver has some extensive superficial
lacerations and is bleeding profusely. The patient becomes progressively more
haemodynamically unstable. Which of the following is the best management option?

Pack the liver and close the abdomen A.

Occlude the hepatic inflow with a pringles manoeuvre and suture the B.
defects

Occlude vascular inflow and resect the most severely affected area C.
anatomically

Perform a portosystemic shunt procedure D.

Suture the defects without vascular occlusion E.

Packing of the liver is the safest option and resection or repair considered later when
the physiology is normalised. Often when the packs are removed all the bleeding has
ceased and the abdomen can be closed without further action. Definitive attempts at
suturing or resection at the primary laparotomy are often complicated by severe
bleeding.

Trauma management

The cornerstone of trauma management is embodied in the principles of ATLS.

Following trauma there is a trimodal death distribution:

 Immediately following injury. Typically as result of brain or high spinal


injuries, cardiac or great vessel damage. Salvage rate is low.
 In early hours following injury. In this group deaths are due to phenomena
such as splenic rupture, sub dural haematomas and haemopneumothoraces
 In the days following injury. Usually due to sepsis or multi organ failure.
Aspects of trauma management

 ABCDE approach.
 Tension pneumothoraces will deteriorate with vigorous ventilation attempts.
 External haemorrhage is managed as part of the primary survey. As a rule
tourniquets should not be used. Blind application of clamps will tend to
damage surrounding structures and packing is the preferred method of
haemorrhage control.
 Urinary catheters and naso gastric tubes may need inserting. Be wary of basal
skull fractures and urethral injuries.
 Patients with head and neck trauma should be assumed to have a cervical
spine injury until proven otherwise.

Thoracic injuries

 Simple pneumothorax
 Mediastinal traversing wounds
 Tracheobronchial tree injury
 Haemothorax
 Blunt cardiac injury
 Diaphragmatic injury
 Aortic disruption
 Pulmonary contusion

Management of thoracic trauma

 Simple pneumothorax insert chest drain. Aspiration is risky in trauma as


pneumothorax may be from lung laceration and convert to tension
pneumothorax.
 Mediastinal traversing wounds These result from situations like stabbings.
Exit and entry wounds in separate hemithoraces. The presence of a mediastinal
haematoma indicates the likelihood of a great vessel injury. All patients should
undergo CT angiogram and oesophageal contrast swallow. Indications for
thoracotomy are largely related to blood loss and will be addressed below.
 Tracheobronchial tree injury Unusual injuries. In blunt trauma most injuries
occur within 4cm of the carina. Features suggesting this injury include
haemoptysis and surgical emphysema. These injuries have a very large air leak
and may have tension pneumothorax.
 Haemothorax Usually caused by laceration of lung vessel or internal
mammary artery by rib fracture. Patients should all have a wide bore 36F chest
drain. Indications for thoracotomy include loss of more than 1.5L blood
initially or ongoing losses of >200ml per hour for >2 hours.
 Cardiac contusions Usually cardiac arrhythmias, often overlying sternal
fracture. Perform echocardiography to exclude pericardial effusions and
tamponade. Risk of arrhythmias falls after 24 hours.
 Diaphragmatic injury Usually left sided. Direct surgical repair is performed.
 Traumatic aortic disruption Commonest cause of death after RTA or falls.
Usually incomplete laceration near ligamentum arteriosum. All survivors will
have contained haematoma. Only 1-2% of patients with this injury will have a
normal chest x-ray.
 Pulmonary contusion Common and lethal. Insidious onset. Early intubation
and ventilation.

Abdominal trauma

 Deceleration injuries are common.


 In blunt trauma requiring laparotomy the spleen is most commonly injured
(40%)
 Stab wounds traverse structures most commonly liver (40%)
 Gunshot wounds have variable effects depending upon bullet type. Small
bowel is most commonly injured (50%)
 Patients with stab wounds and no peritoneal signs up to 25% will not enter the
peritoneal cavity
 Blood at urethral meatus suggests a urethral tear
 High riding prostate on PR = urethral disruption
 Mechanical testing for pelvic stability should only be performed once

Investigations in abdominal trauma

USS Abdominal CT scan Diagnostic


Peritoneal Lavage
Document fluid if Document organ injury Document bleeding if Indication
hypotensive if normotensive hypotensive
Early diagnosis, non Most specific for Early diagnosis and Advantages
invasive and localising injury; 92 to sensitive; 98%
repeatable; 86 to 95% 98% accurate accurate
accurate
Operator dependent Location of scanner Invasive and may Disadvantages
and may miss away from facilities, miss retroperitoneal
retroperitoneal injury time taken for reporting, and diaphragmatic
need for contrast injury

 Amylase may be normal following pancreatic trauma


 Urethrography if suspected urethral injury

A 62 year old woman presents with acute bowel obstruction. She has been vomiting
up to 15 times a day and is taking erythromycin. She suddenly collapses. Her ECG
shows torsades de pointes. What is the management of choice?

IV Atropine A.

IV Potassium B.
IV Magnesium sulphate C.

IV Bicarbonate D.

IV Adrenaline E.
Torsades de pointes: Treatment IV
magnesium sulphate

This woman is likely to have hypokalaemia and hypomagnasaemia as a result of


vomiting. In addition to this, the erythromycin will predispose her to torsades de
pointes. Knowledge of the management of this peri arrest diagnosis is hence
important in surgical practice.

Torsades de pointes

Torsades de pointes ('twisting of the points') is a rare arrhythmia associated with a


long QT interval. It may deteriorate into ventricular fibrillation and hence lead to
sudden death

Causes of long QT interval

 congenital: Jervell-Lange-Nielsen syndrome, Romano-Ward syndrome


 antiarrhythmics: amiodarone, sotalol, class 1a antiarrhythmic drugs
 tricyclic antidepressants
 antipsychotics
 chloroquine
 terfenadine
 erythromycin
 electrolyte: hypocalcaemia, hypokalaemia, hypomagnesaemia
 myocarditis
 hypothermia
 subarachnoid haemorrhage

Management

 IV magnesium sulphate

A 27-yrs-old man sustains a single gunshot wound to the left thigh. In the emergency
department, he is noted to have a large haematoma of his medial thigh. He complains
of parasthesia in his foot. On examination, there are weak pulses palpable distal to the
injury and the patient is unable to move his foot. The appropriate initial management
of this patient is:

Angiography A.

Immediate exploration and repair B.


Fasciotomy of the anterior compartment C.

Observation for resolution of spasm D.

Local wound exploration E.

The five P's of arterial injury include pain, parasthesias, pallor, pulselessness and
paralysis. In the extremities, the tissues most sensitive to anoxia are the peripheral
nerves and striated muscle. The early developments of paresthesias and paralysis are
signals that there is significant ischemia present, and immediate exploration and repair
are warranted. The presence of palpable pulse does not exclude an arterial injury
because this presence may represent a transmitted pulsation through a blood clot.
When severe ischemia is present, the repair must be completed within 6 to 8 h to
prevent irreversible muscle ischemia and loss of limb function. Delay to obtain an
angiogram or to observe for change needlessly prolongs the ischemic time.
Fasciotomy may be required but should be done in conjunction with and after
reestablishment of arterial flow. Local wound exploration is not recommended
because brisk hemorrhage may be encountered without the securing of prior vascular
control.

Vascular trauma

Peripheral and central vessels may be injured by blunt, penetrating or shearing


injuries. Fractures of bones close to vessels may also be associated with vascular
injury or vessel occlusion.

Assessment

 Check for signs of distal perfusion


 Doppler signal distally (monophasic/ biphasic or triphasic)
 Anatomical location (which vessel is likely to be involved)
 Duplex scanning and angiography are "gold standard" tests but may not be
immediately available in the trauma setting

Management

 Almost always operative.


 Obtaining proximal and distal control of affected vessels is crucial.
 Simple lacerations of arteries may be directly closed, or a vein patch applied if
there is a risk of subsequent stenosis.
 Transection of the vessel should be treated by either end to end anastomosis
(often not possible) or an interposition vein graft.
 Use of PTFE in traumatic open injuries will invariably result in infection.

Theme: Paediatric emergencies

Manage conservatively A.
Immediate emergency theatre B.
Treat in emergency department C.
Treat in emergency department under sedation D.
Operate on next emergency list E.

Please select the most appropriate intervention for the scenario given. Each option
may be used once, more than once or not at all.

A 3 year old child inserts a crayon into their external auditory meatus. Attempts 7.
to remove it have not been successful.

You answered Treat in emergency department

The correct answer is Operate on next emergency list

Theme from September 2011 Exam


They would not tolerate removal in the emergency department. The tympanic
membrane should be carefully inspected and again this will be easier under
general anaesthesia.

A 2 year old accidentally inhales a peanut. They arrive in the emergency 8.


department extremely distressed and cyanotic. Imaging shows it to be lodged in
the left main bronchus.

You answered Treat in emergency department under sedation

The correct answer is Immediate emergency theatre

As they are cyanosed it requires immediate removal and this should be


undertaken in a fully staffed theatre. Ideally a rigid bronchoscopy should be
performed.

A 10 year old boy is shot in the head with an airgun pellet. He is concerned that 9.
he will get into trouble and the injury remains concealed for 10 days. Imaging
using CT scanning shows it to be lodged in the frontal lobe.

Manage conservatively

The pellet is small and no serious injury has occurred at this stage. This should
therefore be managed conservatively.

Management of acute cases- Paediatric


 Children will often insert objects into orifices such as the nose and external
auditory meatus
 Assessment includes assessment of airway and haemodynamic status
 Where the airways is not immediately threatened decisions can be made as to
whether to manage in the emergency department or transfer to theatre
 In general children do not tolerate procedures well and it is usually safer to
remove objects in theatre and under general anaesthesia with a secure airway
 A chest x-ray is required to ensure that no object is present in the chest, not all
objects are radiolucent. However, signs such as focal consolidation may
indicate small airway obstruction
 In the case of small bore missile injuries the decision relating to surgery
depends on the size of the missile and its location. Airgun pellets are a
common culprit, if there is a long time interval between the incident and
presentation and the object has not caused any significant problems then it
may be best left alone
 Airgun pellets lodged in the soft tissues (and glass) are usually notoriously
difficult to localise and extract, no matter how superficial. Removal in theatre
is usually the best option. If the object is radiolucent then an image intensifier
should be used
 A 32 year old male is receiving a blood transfusion after being involved in a
road traffic accident. A few minutes after the transfusion he complains of loin
pain. His observations show temperature 39 oC, HR 130bpm and blood
pressure is 95/40mmHg. What is the best test to confirm his diagnosis?

USS abdomen A.

Direct Coomb's test B.

Blood cultures C.

Blood film D.

Sickle cell test E.


Mnemonic for transfusion
reactions:

Got a bad unit

G raft vs. Host disease


O verload
T hrombocytopaenia

A lloimmunization

B lood pressure unstable


A cute haemolytic reaction
D elayed haemolytic reaction
U rticaria
N eutrophilia
I nfection
T ransfusion associated lung
injury

The diagnosis is of an acute haemolytic transfusion reaction, normally due to
ABO incompatibility. Haemolysis of the transfused cells occurs causing the
combination of shock, haemoglobinaemia and loin pain. This may
subsequently lead to disseminated intravascular coagulation. A Coomb's test
should confirm haemolysis. Other tests for haemolysis include: unconjugated
bilirubin, haptoglobin, serum and urine free haemoglobin.

Note that delayed haemolytic reactions are normally associated with


antibodies to the Rh system and occur 5-10 days after transfusion.
 Blood transfusion reactions

Non immune mediated Immune mediated
Hypocalcaemia Pyrexia
CCF Alloimmunization
Infections Thrombocytopaenia
Hyperkalaemia Transfusion associated lung injury
Graft vs Host disease
Urticaria
Acute or delayed haemolysis
ABO incompatibility
Rhesus incompatibility

Notes:
GVHD: lymphocyte proliferation causing organ failure
Transfusion associated lung injury: neutrophil mediated allergic pulmonary
oedema
ABO and Rhesus incompatibility: causes acute haemolytic transfusion
reaction leading to agglutination and haemolysis
 A 32 year old male is receiving a blood transfusion after being involved in a
road traffic accident. A few minutes after the transfusion he complains of loin
pain. His observations show temperature 39 oC, HR 130bpm and blood
pressure is 95/40mmHg. What is the best test to confirm his diagnosis?

USS abdomen A.

Direct Coomb's test B.

Blood cultures C.

Blood film D.
Sickle cell test E.
Mnemonic for transfusion
reactions:

Got a bad unit

G raft vs. Host disease


O verload
T hrombocytopaenia

A lloimmunization

B lood pressure unstable


A cute haemolytic reaction
D elayed haemolytic reaction

U rticaria
N eutrophilia
I nfection
T ransfusion associated lung
injury

The diagnosis is of an acute haemolytic transfusion reaction, normally due to
ABO incompatibility. Haemolysis of the transfused cells occurs causing the
combination of shock, haemoglobinaemia and loin pain. This may
subsequently lead to disseminated intravascular coagulation. A Coomb's test
should confirm haemolysis. Other tests for haemolysis include: unconjugated
bilirubin, haptoglobin, serum and urine free haemoglobin.

Note that delayed haemolytic reactions are normally associated with


antibodies to the Rh system and occur 5-10 days after transfusion.
 Blood transfusion reactions

Non immune mediated Immune mediated
Hypocalcaemia Pyrexia
CCF Alloimmunization
Infections Thrombocytopaenia
Hyperkalaemia Transfusion associated lung injury
Graft vs Host disease
Urticaria
Acute or delayed haemolysis
ABO incompatibility
Rhesus incompatibility

Notes:
GVHD: lymphocyte proliferation causing organ failure
Transfusion associated lung injury: neutrophil mediated allergic pulmonary
oedema
ABO and Rhesus incompatibility: causes acute haemolytic transfusion
reaction leading to agglutination and haemolysis
A 45 year old man complains of sharp chest pain. He is due to have elective surgery
to replace his left hip. He has been bed bound for 3 months. He suddenly collapses;
his blood pressue is 70/40mmHg, heart rate 120 bpm and his saturations are 74% on
air. He is deteriorating in front of you. What is the next best management plan?

Aspirin A.

Thrombolysis with Alteplase B.

Unfractionated heparin C.

Thrombolysis with streptokinase D.

Clopidogrel E.

This man is peri arrest with the diagnosis of pulmonary embolism (chest
pain,bedbound, collapse, low saturations). He needs urgent thrombolysis with
alteplase (he may not survive if you wait for the medical Spr/ITU to arrive!).

Pulmonary embolism: management

A summary of the British Thoracic Society guidelines

 Heparin should be given if intermediate or high clinical probability before


imaging.
 Unfractionated heparin (UFH) should be considered (a) as a first dose bolus,
(b) in massive PE, or (c) where rapid reversal of effect may be needed.
 Otherwise, low molecular weight heparin (LMWH) should be considered as
preferable to UFH, having equal efficacy and safety and being easier to use.
 Oral anticoagulation should only be commenced once VTE has been reliably
confirmed.
 The target INR should be 2.0-3.0; when this is achieved, heparin can be
discontinued.
 The standard duration of oral anticoagulation is: 46 weeks for temporary risk
factors, 3 months for first idiopathic, and at least 6 months for other; the risk
of bleeding should be balanced with that of further VTE.

Massive PE

 CTPA or echocardiography will reliably diagnose clinically massive PE.


 Thrombolysis is 1st line for massive PE (ie circulatory failure) and may be
instituted on clinical grounds alone if cardiac arrest is imminent; a 50 mg
bolus of alteplase is recommended.
 Invasive approaches (thrombus fragmentation and IVC filter insertion) should
be considered where facilities and expertise are readily available.
A 30 year old woman, who is 30 weeks pregnant, attends the varicose vein clinic. She
suddenly complains of shortness of breath and chest pain. She has no underlying lung
condition. Her saturations are 92 % air, blood pressure 150/80 mmHg and her chest
sounds clear. What is the main investigation of choice to confirm her diagnosis?

Lung spirometry A.

Half dose scintigraphy B.

CTPA C.

CXR D.

Full dose scintigraphy E.

The main differential diagnosis is pulmonary embolism. CXR should be performed


first in second and third trimester to exclude other diagnoses such as pneumothorax or
pneumonia. Concerns surrounding radiation exposure have been discounted at this
stage, as the need to establish a diagnosis is the major priority. If the chest x-ray is
normal, then half dose scintigraphy or CTPA (if the patient has underlying lung
pathology) is performed.

Reference:

Scarsbrook A.Fand Gleeson V. Investigating suspected pulmonary embolism in


pregnancy. BMJ 2003 (326) : 1135 doi: 10.1136/bmj.7399.1135.

Chest pain in pregnancy

Aortic dissection

 Predisposing factors in pregnancy are hypertension, congenital heart disease


and Marfan's syndrome
 Mainly Stanford type A dissections
 Sudden tearing chest pain, transient syncope
 Patient may be cold and clammy, hypertensive and have an aortic
regurgitation murmur
 Involvement of the right coronary artery may cause inferior myocardial
infarction

Surgical management
Management Gestational
timeframe
Aortic repair with the fetus kept in utero < 28/40
Dependent on fetal condition 28-32/40
Primary Cesarean section followed by aortic repair at the same > 32/40
operation
Mitral stenosis

 Most cases associated with rheumatic heart disease


 Becoming less common in British women; suspect in Immigrant women
 Commonest cardiac condition in pregnancy
 Commonly associated with mortality
 Valve surgery; balloon valvuloplasty preferable

Pulmonary embolism

 Leading cause of mortality in pregnancy


 Half dose scintigraphy; CT chest if underlying lung disease should aid
diagnosis
 Treatment with low molecular weight heparin throughout pregnancy and 4-6
weeks after childbirth
 Warfarin is contra indicated in pregnancy

References
1. Bates S.M. and Ginsberg J.S. How we manage venous thromboembolism during
pregnancy. Blood 2002 (100): 3470-3478.

2. Scarsbrook A.Fand Gleeson V. Investigating suspected pulmonary embolism in


pregnancy. BMJ 2003 (326) : 1135 doi: 10.1136/bmj.7399.1135.

3. Morley C. A. and Lim B. A. Lesson of the Week: The risks of delay in diagnosis of
breathlessness in pregnancy. BMJ 1995 (311) : 1083.

A 30 year old woman, who is 30 weeks pregnant, attends the varicose vein clinic. She
suddenly complains of shortness of breath and chest pain. She has no underlying lung
condition. Her saturations are 92 % air, blood pressure 150/80 mmHg and her chest
sounds clear. What is the main investigation of choice to confirm her diagnosis?

Lung spirometry A.

Half dose scintigraphy B.

CTPA C.

CXR D.

Full dose scintigraphy E.

The main differential diagnosis is pulmonary embolism. CXR should be performed


first in second and third trimester to exclude other diagnoses such as pneumothorax or
pneumonia. Concerns surrounding radiation exposure have been discounted at this
stage, as the need to establish a diagnosis is the major priority. If the chest x-ray is
normal, then half dose scintigraphy or CTPA (if the patient has underlying lung
pathology) is performed.

Reference:

Scarsbrook A.Fand Gleeson V. Investigating suspected pulmonary embolism in


pregnancy. BMJ 2003 (326) : 1135 doi: 10.1136/bmj.7399.1135.

Chest pain in pregnancy

Aortic dissection

 Predisposing factors in pregnancy are hypertension, congenital heart disease


and Marfan's syndrome
 Mainly Stanford type A dissections
 Sudden tearing chest pain, transient syncope
 Patient may be cold and clammy, hypertensive and have an aortic
regurgitation murmur
 Involvement of the right coronary artery may cause inferior myocardial
infarction

Surgical management
Management Gestational
timeframe
Aortic repair with the fetus kept in utero < 28/40
Dependent on fetal condition 28-32/40
Primary Cesarean section followed by aortic repair at the same > 32/40
operation

Mitral stenosis

 Most cases associated with rheumatic heart disease


 Becoming less common in British women; suspect in Immigrant women
 Commonest cardiac condition in pregnancy
 Commonly associated with mortality
 Valve surgery; balloon valvuloplasty preferable

Pulmonary embolism

 Leading cause of mortality in pregnancy


 Half dose scintigraphy; CT chest if underlying lung disease should aid
diagnosis
 Treatment with low molecular weight heparin throughout pregnancy and 4-6
weeks after childbirth
 Warfarin is contra indicated in pregnancy

References
1. Bates S.M. and Ginsberg J.S. How we manage venous thromboembolism during
pregnancy. Blood 2002 (100): 3470-3478.

2. Scarsbrook A.Fand Gleeson V. Investigating suspected pulmonary embolism in


pregnancy. BMJ 2003 (326) : 1135 doi: 10.1136/bmj.7399.1135.

3. Morley C. A. and Lim B. A. Lesson of the Week: The risks of delay in diagnosis of
breathlessness in pregnancy. BMJ 1995 (311) : 1083.

Theme: Bleeding disorders

Vitamin K deficiency A.
von Willebrand's disease B.
Acquired haemophilia C.
Haemophilia B D.
Protein C deficiency E.
Disseminated intravascular coagulation F.
Factor V Leiden G.
Excess heparin H.
Warfarin overdose I.

What is the most likely diagnosis for the scenario given? Each option may be used
once, more than once or not at all.

A 33 year old female is admitted for varicose vein surgery. She is fit and well. 13.
After the procedure she is persistently bleeding. She is known to have
menorrhagia. Investigations show a prolonged bleeding time and increased
APTT. She has a normal PT and platelet count.

von Willebrand's disease

Bleeding post operatively, epistaxis and menorrhagia may indicate a diagnosis


of vWD. Haemoarthroses are rare. The bleeding time is usually normal in
haemophilia (X-linked) and vitamin K deficiency.

A 70 year old coal miner presents with 3 weeks of haematuria and bruising. He 14.
is normally fit and well. He is on no medications. His results reveal:
Hb 9.0
WCC 11
Pl 255
PT 16 (normal)
APTT 58 (increased)
Thrombin time 20 (normal).

You answered Protein C deficiency


The correct answer is Acquired haemophilia

Factor 8 acquires an inhibitor. The elderly, pregnancy, malignancy and


autoimmune conditions are associated with acquired haemophilia. Prolonged
APTT is key to the diagnosis. Management involves steroids.

A 33 year old female is attends the day unit for elective varicose vein surgery. 15.
She has previously had recurrent pulmonary embolic events. After the
procedure she is persistently bleeding. Her APTT is 52 (increased).

You answered Warfarin overdose

The correct answer is Factor V Leiden

A combination of thromboembolism and bleeding in a young woman should


raise the possibility of antiphospholipid syndrome. Other features may include
foetal loss, venous and arterial thrombosis and thrombocytopenia. Protein C is
not associated with a prolonged APTT.

Theme from January 2012 exam

Abnormal coagulation

Factors affected Cause


Prevents activation factors 2,9,10,11 Heparin
Affects synthesis of factors 2,7,9,10 Warfarin
Factors 1,2,5,8,11 DIC
Factors 1,2,5,7,9,10 Liver disease

Interpretation blood clotting test results


Bleeding time PT APTT Disorder
Normal Normal Increased Haemophilia
Increased Normal Increased von Willebrand's disease
Normal Increased Increased Vitamin K deficiency
A 54-year-old man is brought to the Emergency Department after being found
collapsed in the street. He is known to have a history of alcoholic liver disease. Blood
tests reveal the following:

1.62 mmol/l Calcium


33 g/l Albumin
Which one of the following is the most appropriate management of the calcium
result?

10ml of 10% calcium chloride over 10 minutes A.

20% albumin infusion B.

10ml of 10% calcium gluconate over 10 minutes C.

No action D.

10ml of 10% calcium chloride over 4 hours E.

Even after correction for the low albumin level this patient has significant
hypocalcaemia which should be corrected.

Hypocalcaemia: causes and management

The clinical history combined with parathyroid hormone levels will reveal the cause
of hypocalcaemia in the majority of cases

Causes

 Vitamin D deficiency (osteomalacia)


 Acute pancreatitis
 Chronic renal failure
 Hypoparathyroidism (e.g. post thyroid/parathyroid surgery)
 Pseudohypoparathyroidism (target cells insensitive to PTH)
 Rhabdomyolysis (initial stages)
 Magnesium deficiency (due to end organ PTH resistance)

Management

 Acute management of severe hypocalcaemia is with intravenous replacement.


The preferred method is with intravenous calcium gluconate, 10ml of 10%
solution over 10 minutes
 Intravenous calcium chloride is more likely to cause local irritation
 ECG monitoring is recommended
 Further management depends on the underlying cause

A 25 year old male pedestrian is hit by a van on a busy road. He is brought to the
Emergency Department by ambulance. On examination he is dyspneoic, and hypoxic
despite administration of high flow 100% oxygen. His blood pressure is 110/70 and
pulse rate is 115 bpm. The right side of his chest is hyper-resonant on percussion and
has decreased breath sounds. The trachea is deviated to the left. What is the most
likely underlying diagnosis?

Fat embolism A.

Tension pneumothorax B.

Rupture of the right main bronchus C.

Rupture of the diaphragm D.

Pulmonary contusion E.

Blunt or penetrating chest trauma that creates a flap type defect on the surface of the
lung can result in a tension pneumothorax. Typical features include dyspnoea,
progressive hypoxia, hyperresonance and tracheal deviation. Treatment is with needle
decompression and chest tube insertion.

Thoracic trauma

Types of thoracic trauma

 Often laceration to lung parenchyma with flap Tension


 Pressure develops in thorax pneumothorax
 Most common cause is mechanical ventilation in patient
with pleural injury
 Symptoms overlap with cardiac tamponade, hyper-
resonant percussion note is more likely in tension
pnemothorax

 Chest wall disconnects from thoracic cage Flail chest


 Multiple rib fractures (at least two fractures per rib in at
least two ribs)
 Associated with pulmonary contusion
 Abnormal chest motion
 Avoid over hydration and fluid overload

 Most common cause is lung laceration with air leakage Pneumothorax


 Most traumatic pneumothoraces should have a chest
drain
 Patients with traumatic pneumothorax should never be
mechanically ventilated until a chest drain is inserted

 Most commonly due to laceration of lung, intercostal Haemothorax


vessel or internal mammary artery
 Haemothoraces large enough to appear on CXR are
treated with large bore chest drain
 Surgical exploration is warranted if >1500ml blood
drained immediately

 Beck's triad: elevated venous pressure, reduced arterial Cardiac tamponade


pressure, reduced heart heart sounds.
 Pulsus paradoxus
 May occur with as little as 100ml blood

 Most common potentially lethal chest injury Pulmonary


 Arterial blood gases and pulse oximetry important contusion
 Early intubation within an hour if significant hypoxia

 Usually occurs secondary to chest wall injury Blunt cardiac injury


 ECG may show features of myocardial infarction
 Sequelae: hypotension, arrhythmias, cardiac wall motion
abnormalities

 Deceleration injuries Aorta disruption


 Contained haematoma
 Widened mediastinum

 Most due to motor vehicle accidents and blunt trauma Diaphragm


causing large radial tears (laceration injuries result in disruption
small tears)
 More common on left side
 Insert gastric tube, which will pass into the thoracic
cavity

 Entrance wound in one haemothorax and exit Mediastinal


wound/foreign body in opposite haemothorax traversing wounds
 Mediastinal haematoma or pleural cap suggests great
vessel injury
 Mortality is 20%

References
ATLS Manual 8th Edition
Theme: Management of skin injuries

Wound excision and primary closure A.


Simple primary closure B.
Delayed primary closure C.
Debridement and healing by secondary intention D.
Split thickness skin graft E.
Full thickness skin graft F.
Free flap G.
Pedicled flap H.
For the injuries described please select the most appropriate management. Each option
may be used once, more than once or not at all.

A 25 year old man is playing with a Pit Bull terrier which bites off a 18.
substantial portion of his nose.

You answered Free flap

The correct answer is Debridement and healing by secondary intention

Theme from April 2012 Exam


Dog bites are an extremely dirty form of wound and as a result may be most
safely managed by being left to heal by secondary intention. The cosmetic
results of this strategy will be significant and major reconstructive surgery will
be needed at a later stage. Some plastic surgeons will attempt a debridement
and primary repair with either a pedicled flap or full thickness graft. The
traditional teaching would be for secondary intention healing.

A 7 year old boy falls over and sustains a 6cm laceration to his head. On 19.
inspection his wound contains some dirt in it.

You answered Delayed primary closure

The correct answer is Wound excision and primary closure

By debriding the wound, the area can then be primarily closed. Prophylactic
antibiotics should be administered.

A 45 year old man is gardening is puts a fork into his foot. On examination 20.
there are cutaneous defects and the surrounding skin looks dusky.

Debridement and healing by secondary intention

The skin changes described here should be debrided. Closure would not be
safe with the skin changes documented and the wound should be left open.

Methods of wound closure

Indication Method of
closure
 Clean wound, usually surgically created or following minor Primary closure
trauma
 Standard suturing methods will usually suffice
 Wound heals by primary intention

 Similar methods of actual closure to primary closure Delayed primary


 May be used in situations where primary closure is either not closure
achievable or not advisable e.g. infection

 Uses negative pressure therapy to facilitate wound closure Vacuum assisted


 Sponge is inserted into wound cavity and then negative closure
pressure applied
 Advantages include removal of exudate and versatility
 Disadvantages include cost and risk of fistulation if used
incorrectly on sites such as bowel

 Superficial dermis removed with Watson knife or dermatome Split thickness


(commonly from thigh) skin grafts
 Remaining epithelium regenerates from dermal appendages
 Coverage may be increased by meshing

 Whole dermal thickness is removed Full thickness


 Sub dermal fat is then removed and graft placed over donor skin grafts
site
 Better cosmesis and flexibility at recipient site
 Donor site "cost"

 Viable tissue with a blood supply Flaps


 May be pedicled or free
 Pedicled flaps are more reliable, but limited in range
 Free flaps have greater range but carry greater risk of
breakdown as they require vascular anastomosis

A 19 year old student is involved in a head on car collision. He complains of severe


chest pain. A Chest x-ray performed as part of a trauma series shows widening of the
mediastinum. Which is the most likely injury in this scenario?

Rupture of the distal oesophagus A.

Rupture of the left main bronchus B.

Rupture of the aorta proximal to the left subclavian artery C.

Rupture of the aorta distal to the left subclavian artery D.

Rupture of the inferior vena cava E.

The aorta may be injured in deceleration accidents. In the setting of deceleration


injury, chest pain and mediastinal widening the most likely problem is aortic rupture.
This will typically occur distal to the left subclavian artery. Rupture of the proximal
aorta may occur. However, survival is unlikely. It is important to note that the
question uses the term Most likely injury as this is the component that distinguishes an
ascending rupture from a descending rupture.

Thoracic aorta rupture

 Mechanism of injury: Decelerating force i.e. RTA, fall from a great height
 Most people die at scene
 Survivors may have an incomplete laceration at the ligamentum arteriosum of
the aorta.

Clinical features

 Contained haematoma: persistent hypotension


 Detected mainly by history, CXR changes

CXR changes

 Widened mediastinum
 Trachea/Oesophagus to right
 Depression of left main stem bronchus
 Widened paratracheal stripe/paraspinal interfaces
 Space between aorta and pulmonary artery obliterated
 Rib fracture/left haemothorax

Diagnosis
Angiography, usually CT aortogram.

Treatment
Repair or replacement. Ideally they should undergo endovascular repair.
A 19 year old student is involved in a head on car collision. He complains of severe
chest pain. A Chest x-ray performed as part of a trauma series shows widening of the
mediastinum. Which is the most likely injury in this scenario?

Rupture of the distal oesophagus A.

Rupture of the left main bronchus B.

Rupture of the aorta proximal to the left subclavian artery C.

Rupture of the aorta distal to the left subclavian artery D.

Rupture of the inferior vena cava E.

The aorta may be injured in deceleration accidents. In the setting of deceleration


injury, chest pain and mediastinal widening the most likely problem is aortic rupture.
This will typically occur distal to the left subclavian artery. Rupture of the proximal
aorta may occur. However, survival is unlikely. It is important to note that the
question uses the term Most likely injury as this is the component that distinguishes an
ascending rupture from a descending rupture.

Thoracic aorta rupture

 Mechanism of injury: Decelerating force i.e. RTA, fall from a great height
 Most people die at scene
 Survivors may have an incomplete laceration at the ligamentum arteriosum of
the aorta.

Clinical features

 Contained haematoma: persistent hypotension


 Detected mainly by history, CXR changes

CXR changes

 Widened mediastinum
 Trachea/Oesophagus to right
 Depression of left main stem bronchus
 Widened paratracheal stripe/paraspinal interfaces
 Space between aorta and pulmonary artery obliterated
 Rib fracture/left haemothorax

Diagnosis
Angiography, usually CT aortogram.

Treatment
Repair or replacement. Ideally they should undergo endovascular repair.
Theme: Management of head and neck trauma

Observation A.
CT head within 1h B.
CT head within 8h C.
Urgent neurosurgical review (even before CT head performed) D.
3 view c-spine xray E.
2 view c-spine xray F.
CT c-spine G.

What is the best initial management plan for the injuries described? Each option may
be used once, more than once or not at all.
A 22 year old mechanic is involved in a fight. He is hit on the head with a 22.
hammer. On examination he had clinical evidence of an open depressed skull
fracture and a GCS of 6/15.

Urgent neurosurgical review (even before CT head performed)

A patient with GCS <8 or = to 8 needs urgent neurosurgical review. Especially


when an open fracture is present.

A 67 year old retired lawyer falls down the stairs. His GCS is 15/15 and he has 23.
some bruising over the mastoid.

You answered Observation

The correct answer is CT head within 1h

This patient has a basal skull fracture, which is indicated by a positive Battle's
sign. He should have a CT head within 1h.

A 52 year old secretary falls down the stairs. She complains of neck pain. She 24.
has a GCS of 15/15 and no neurology. She is unable to rotate her c-spine 45
degrees to the left and right.

You answered 2 view c-spine xray

The correct answer is 3 view c-spine xray

She should have her c-spine immobilised and a 3 view c-spine xray.

Head injury management- NICE Guidelines

Summary of guidelines

 All patients should be assessed within 15 minutes on arrival to A&E


 Document all 3 components of the GCS
 If GCS <8 or = to 8, consider stabilising the airway
 Treat pain with low dose IV opiates (if safe)
 Full spine immobilisation until assessment if:

- GCS < 15
- neck pain/tenderness
- paraesthesia extremities
- focal neurological deficit
- suspected c-spine injury

If a c-spine injury is suspected a 3 view c-spine x-ray is indicated. CT c-spine is


preferred if:
- Intubated
- GCS <13
- Normal x-ray but continued concerns regarding c-spine injury

Immediate CT head (within 1h) if:

 GCS < 13 on admission


 GCS < 15 2h after admission
 Suspected open or depressed skull fracture
 Suspected skull base fracture (panda eyes, Battle's sign, CSF from nose/ear,
bleeding ear)
 Focal neurology
 Vomiting > 1 episode
 Post traumatic seizure
 Coagulopathy

Contact neurosurgeon if:

 Persistent GCS < 8 or = 8


 Unexplained confusion > 4h
 Reduced GCS after admission
 Progressive neurological signs
 Incomplete recovery post seizure
 Penetrating injury
 Cerebrospinal leak

Observations

 1/2 hourly GCS until 15

Reference
http://guidance.nice.org.uk/CG56/QuickRefGuide/pdf/English
A 28 year old African man is admitted with acute severe abdominal pain. He has just
flown into the UK long haul and the pain developed whilst in flight. On examination
he is tender in the left upper quadrant. His blood tests are as shown.

Hb 6 g/dl
Reticulocyte count 15%.
Ultrasound shows a spleen with a heterogeous texture and a few small gallstones but
is otherwise normal.
What is the most likely diagnosis?
Pancreatitis A.

Parvovirus infection B.

Sickle cell anaemia C.

Pulmonary embolism D.

Beta Thalassaemia minor E.

A combination of a high reticulocyte count and severe anaemia indicates sickle cell
anaemia, however another differential can be of a transient aplastic crisis due to
parvovirus. This is less likely as this causes a reticulocytopenia rather than a
reticulocytosis.

Parvovirus B19 infects erythroid progenitor cells in the bone marrow and causes
temporary cessation of red blood cell production, patients who have underlying
hematologic abnormalities are at risk of cessation of red blood cell production if they
become infected. This can result in a transient aplastic crisis. Thus, patients with
sickle cell anaemia are at risk. Typically, these patients have a viral prodrome
followed by anaemia, often with haemoglobin concentrations falling below 5.0 g/dL
and reticulocytosis.

Sickle cell anaemia

 Autosomal recessive
 Single base mutation
 Deoxygenated cells become sickle in shape
 Causes: short red cell survival, obstruction of microvessels and infarction
 Sickling is precipitated by: dehydration, infection, hypoxia
 Manifest at 6 months age
 Africans, Middle East, Indian
 Diagnosis: Hb electrophoresis

Sickle crises

 Bone pain
 Pleuritic chest pain: acute sickle chest syndrome commonest cause of death
 CVA, seizures
 Papillary necrosis
 Splenic infarcts
 Priapism
 Hepatic pain

Hb does not fall during a crisis, unless there is


 Aplasia: parvovirus
 Acute sequestration
 Haemolysis

Long-term complications

 Infections: Streptococcus pnemoniae


 Chronic leg ulcers
 Gallstones: haemolysis
 Aseptic necrosis of bone
 Chronic renal disease
 Retinal detachment, proliferative retinopathy

Surgical complications

 Bowel ischaemia
 Cholecystitis
 Avascular necrosis

Management

 Supportive
 Hydroxyurea
 Repeated transfusions pre operatively
 Exchange transfusion in emergencies

Sickle cell trait

 Heterozygous state
 Asymptomatic
 Symptoms associated with extreme situations ie anaesthesia complications
 Protective against Plasmodium falciparum
 Theme: Blood transfusion reactions

Neutrophilic febrile reaction A.


Acute haemolytic transfusion reaction B.
Delayed haemolytic transfusion reaction C.
Pulmonary oedema D.
Sickle cell crisis E.
Transfusion associated lung injury F.
Graft vs. Host disease G.

Please select the most likely underlying cause for each scenario. Each option
may be used once, more than once or not at all.
A 22 year old man is having a blood transfusion after losing blood from 2.
haemorrhoids. He is normally fit and well. After 4h, during the transfusion, he
complains of sudden onset shortness of breath and chest pain. On examination
his temperature is 37.2, saturations are 88% on air, blood pressure 100/55
mmHg and HR 110 bpm. He has crepitations bilaterally up to the midzones. He
is given IV frusemide, but deteriorates and is admitted to ITU. A pulmonary
catheter is inserted and the PCWP is 10 mmHg.

You answered Pulmonary oedema

The correct answer is Transfusion associated lung injury

The pulmonary catheter reading indicates that this is not a case of fluid overload
(the PCWP should be high, normal values PCWP systolic 7mmHg, diastolic
10mmHg). Transfusion associated lung injury is a rare reaction causing
neutrophilic mediated allergic pulmonary oedema. Patient's have antibodies to
donor leukocytes. It is important to consider this as a diagnosis when patients
don't respond to treatment for pulmonary oedema. Patients normally respond to
supportive therapy including fluids and oxygen.

A 32 year male with leukaemia attends the day unit for a blood transfusion. Five 3.
days after the transfusion he attends A&E with a temperature of 38.5,
erythroderma and desquamation.

You answered Delayed haemolytic transfusion reaction

The correct answer is Graft vs. Host disease

This is associated with transfusion of unirradiated blood in immunosupressed


patients. Transfusion associated GVHD can occur 4-30 days after a transfusion
and follows a sub acute pathway. Patients may also have diarrhoea and
abnormal liver function tests. Management involves steroid therapy.

A 22 year old man is having a blood transfusion after losing blood from 4.
haemorrhoids. He is normally fit and well. 3h during the transfusion he
complains of sudden onset abdominal pain and nausea. His temperature is 39
degrees, Blood pressure 98/42 mmHg, HR 105 bpm and saturations 94% air.
His urine appears dark.

Acute haemolytic transfusion reaction

Rapid intravascular haemolysis leading to shock, DIC and death can occur with
this reaction.

 Blood transfusion reactions

Non immune mediated Immune mediated


Hypocalcaemia Pyrexia
CCF Alloimmunization
Infections Thrombocytopaenia
Hyperkalaemia Transfusion associated lung injury
Graft vs Host disease
Urticaria
Acute or delayed haemolysis
ABO incompatibility
Rhesus incompatibility

Notes:
GVHD: lymphocyte proliferation causing organ failure
Transfusion associated lung injury: neutrophil mediated allergic pulmonary
oedema
ABO and Rhesus incompatibility: causes acute haemolytic transfusion
reaction leading to agglutination and haemolysis
A 49-year-old male sustained a severe blunt injury just below the bridge of the nose
with industrial machinery. Imaging demonstrates a fracture involving the superior
orbital fissure. On examination an ipsilateral pupillary defect is present and loss of the
corneal reflexes. In addition to these examination findings, which of the following
will not be present?

Altered cutaneous sensation from the forehead to the vertex A.

Ptosis B.

Complete opthalmoplegia C.

Nystagmus D.

Enopthalmos E.
Orbital apex syndrome
This is an extension of superior orbital fissure syndrome and includes compression of
the optic nerve passing through the optic foramen. It is indicated by features of
superior orbital fissure syndrome and ipsilateral afferent pupillary defect.

This type of injury will result in the orbital apex syndrome (See above). As such
opthalmoplegia will be present and nystagmus cannot occur.

Craniomaxillofacial injuries

Craniomaxillofacial injuries in the UK are due to:

 Interpersonal violence (52%)


 Motor vehicle accidents (16%)
 Sporting injuries (19%)
 Falls (11%)

Le Fort Fractures
Feature Grade
The fracture extends from the nasal septum to the lateral pyriform rims, travels Le Fort
horizontally above the teeth apices, crosses below the zygomaticomaxillary 1
junction, and traverses the pterygomaxillary junction to interrupt the pterygoid
plates.
These fractures have a pyramidal shape and extend from the nasal bridge at or Le Fort
below the nasofrontal suture through the frontal process of the maxilla, 2
inferolaterally through the lacrimal bones and inferior orbital floor and rim
through or near the inferior orbital foramen, and inferiorly through the anterior
wall of the maxillary sinus; it then travels under the zygoma, across the
pterygomaxillary fissure, and through the pterygoid plates.
These fractures start at the nasofrontal and frontomaxillary sutures and extend Le Fort
posteriorly along the medial wall of the orbit through the nasolacrimal groove 3
and ethmoid bones. The thicker sphenoid bone posteriorly usually prevents
continuation of the fracture into the optic canal. Instead, the fracture continues
along the floor of the orbit along the inferior orbital fissure and continues
superolaterally through the lateral orbital wall, through the zygomaticofrontal
junction and the zygomatic arch. Intranasally, a branch of the fracture extends
through the base of the perpendicular plate of the ethmoid, through the vomer,
and through the interface of the pterygoid plates to the base of the sphenoid.
This type of fracture predisposes the patient to CSF rhinorrhea more
commonly than the other types.

Ocular injuries
Superior orbital fissure syndrome
Severe force to the lateral wall of the orbit resulting in compression of neurovascular
structures. Results in :

 Complete opthalmoplegia and ptosis (Cranial nerves 3, 4, 6 and nerve to


levator palpebrae superioris)
 Relative afferent pupillary defect
 Dilatation of the pupil and loss of accommodation and corneal reflexes
 Altered sensation from forehead to vertex (frontal branch of trigeminal nerve)

Orbital blow out fracture


Typically occurs when an object of slightly larger diameter than the orbital rim strikes
the incompressible eyeball. The bone fragment is displaced downwards into the antral
cavity, remaining attached to the orbital periosteum. Periorbital fat may be herniated
through the defect, interfering with the inferior rectus and inferior oblique muscles
which are contained within the same fascial sheath. This prevents upward movement
and outward rotation of the eye and the patient experiences diplopia on upward gaze.
The initial bruising and swelling may make assessment difficult and patients should
usually be reviewed 5 days later. Residual defects may require orbital floor
reconstruction.
Nasal Fractures

 Common injury
 Ensure new and not old deformity
 Control epistaxis
 CSF rhinorrhoea implies that the cribriform plate has been breached and
antibiotics will be required.
 Usually best to allow bruising and swelling to settle and then review patient
clinically. Major persistent deformity requires fracture manipulation, best
performed within 10 days of injury.

Retrobulbar haemorrhage
Rare but important ocular emergency. Presents with:

 Pain (usually sharp and within the globe)


 Proptosis
 Pupil reactions are lost
 Paralysis (eye movements lost)
 Visual acuity is lost (colour vision is lost first)

May be the result of Le Fort type facial fractures.

Management:

 Mannitol 1g/Kg as 20% infusion, Osmotic diuretic, Contra-indicated in


congestive heart failure and pulmonary oedema
 Acetazolamide 500mg IV, (Monitor FBC/U+E) Reduces aqueous pressure by
inhibition of carbonic anhydrase (used in glaucoma)
 Dexamethasone 8mg orally or intravenously
 In a traumatic setting an urgent catholysis may be needed prior to definitive
surgery.

Consider
Papaverine 40mg smooth muscle relaxant
Dextran 40 500mls IV improves perfusion

Which option is not recommended during the management of compartment


syndrome?

Anticoagulation A.

Keep limb level with the body B.

Intravenous fluids C.

Pain control D.
Fasciotomy E.

Anticoagulation will worsen compartment syndrome.

Compartment syndrome

 This is a particular complication that may occur following fractures (or


following ischaemia reperfusion injury in vascular patients). It is characterised
by raised pressure within a closed anatomical space.
 The raised pressure within the compartment will eventually compromise tissue
perfusion resulting in necrosis. The two main fractures carrying this
complication include supracondylar fractures and tibial shaft injuries.

Symptoms and signs

 Pain, especially on movement (even passive)


 Parasthesiae
 Pallor may be present
 Arterial pulsation may still be felt as the necrosis occurs as a result of
microvascular compromise
 Paralysis of the muscle group may occur

Diagnosis

 Is made by measurement of intracompartmental pressure measurements.


Pressures in excess of 20mmHg are abnormal and >40mmHg is diagnostic.

Treatment

 This is essentially prompt and extensive fasciotomies


 In the lower limb the deep muscles may be inadequately decompressed by the
inexperienced operator when smaller incisions are performed
 Myoglobinuria may occur following fasciotomy and result in renal failure and
for this reason these patients require aggressive IV fluids
 Where muscle groups are frankly necrotic at fasciotomy they should be
debrided and amputation may have to be considered
 Death of muscle groups may occur within 4-6 hours

Theme: Complications of burns

Deep vein thrombosis A.


Curlings Ulcer B.
Contracture C.
Type I respiratory failure D.
Type II respiratory failure E.
Toxic shock syndrome F.
Compartment syndrome G.
Rhabdomyolysis H.
Disseminated intravascular coagulation I.

For each clinical scenario please select the most likely complication to have occurred.
Each option may be used once, more than once or not at all.

A 10 year old child is admitted with severe 30% burns following a house fire. 7.
After wound cleaning and dressings he is admitted to critical care. 1 day
following skin grafts he becomes tachycardic and hypotensive. He vomits twice
and this shows evidence of haematemesis

Curlings Ulcer

Stress ulcers may occur in the duodenum of burns patients and are more
common in children.

A 26 year old electrician suffers a full thickness high voltage burn to his leg. On 8.
routine urine analysis he has + blood. His U+E's show mild hyperkalaemia and
a CK of 3000

Rhabdomyolysis

Electrical high voltage burns are associated with rhabdomyolysis. Acute tubular
necrosis may occur. Aggressive IV fluids should be given

A 45 year old man is admitted after his clothing caught fire. He suffers a full 9.
thickness circumferential burn to his lower thigh. He complains of increasing
pain in lower leg and on examination there is parasthesia and severe pain in the
lower leg. Foot pulses are normal

Compartment syndrome

Circumferential burns may constrict the limb and cause a compartment


syndrome to develop. Eshcarotomy is required, and compartmental
decompression.

Burns

Types of burn
Management Blanching Skin Skin layers Type of burn
appearance affected
Yes Red, moist Epidermis Epidermal/Superficial
Normally heals Yes Pale, dry Epidermis and Superficial partial
with no part of papillary thickness
intervention dermis affected
Needs surgical No Mottled red Epidermis, whole Deep partial thickness
intervention colour papillary dermis
(depending on affected
site)
Burns centre No Dry, leathery Whole skin layer Full thickness
hard wound and subcutaneous
tissue affected

Depth of burn assessment

 Bleeding on needle prick


 Sensation
 Appearance
 Blanching to pressure

Percentage burn estimation


Lund Browder chart: most accurate even in children
Wallace rule of nines
Palmar surface: surface area palm = 0.8% burn

>15% body surface area burns in adults needs urgent burn fluid resuscitation

Transfer to burn centre if:

 Need burn shock resuscitation


 Face/hands/genitals affected
 Deep partial thickness or full thickness burns
 Significant electrical/chemical burns

Escharotomies

 Indicated in circumferential full thickness burns to the torso or limbs.


 Careful division of the encasing band of burn tissue will potentially improve
ventilation (if the burn involves the torso), or relieve compartment syndrome
and oedema (where a limb is involved)

References
www.euroburn.org/e107files/downloads/guidelinesburncare.pdf
Hettiaratchy S & Papini R. Initial management of a major burn: assessment and
resuscitation. BMJ 2004;329:101-103
A 23 year old man who plays rugby for a hobby presents with recurrent anterior
dislocation of the shoulder. Which of the following abnormalities is most likely to be
present to account for this?

Rotator cuff tear A.

Biceps tendon rupture B.

Bankart lesion C.

Axillary nerve injury D.

Infraspinatus tendinitis E.

Anterior dislocations are the most common. When recurrent a Bankart lesion is the
most common underlying abnormality. This is usually visualised by CT and MRI
scanning and often repaired arthroscopically.

Shoulder disorders

Proximal humerus fractures

 Very common. Usually through the surgical neck. Number of classification


systems though for practical purposes describing the number of fracture
fragments is probably easier. Some key points:
 It is rare to have fractures through the anatomical neck.
 Anatomical neck fractures which are displaced by >1cm carry a risk of
avascular necrosis to the humeral head.
 In children the commonest injury pattern is a greenstick fracture through the
surgical neck.
 Impacted fractures of the surgical neck are usually managed with a collar and
cuff for 3 weeks followed by physiotherapy.
 More significant displaced fractures may require open reduction and fixation
or use of an intramedullary device.

Types of shoulder dislocation

 Glenohumeral dislocation (commonest): anterior shoulder dislocation most


common
 Acromioclavicular dislocation (12%): clavicle loses all attachment with the
scapula
 Sternoclavicular dislocation (uncommon)

Types of glenohumeral dislocation:


 External rotation and abduction Anterior shoulder
 35-40% recurrent (it is the commonest disorder) dislocation
 Assocociated with greater tuberosity fracture, Bankart
lesion, Hill-Sachs defect

Luxatio erecta Inferior shoulder


dislocation
 Proportion misdiagnosed. Posterior shoulder
 Rim's sign, light bulb sign. dislocation
 Assocociated with Trough sign

Rare and usually follow major trauma. Superior shoulder


dislocation

Treatment
Prompt reduction is the mainstay of treatment and is usually performed in the
emergency department. Neurovascular status must be checked pre and post reduction
and x-rays should be performed again post reduction to ensure no fracture has
occurred. In recurrent anterior dislocation there is usually a Bankart lesion and this
may be repaired surgically. Recurrent posterior dislocations may be repaired in a
similar manner to anterior lesions but using a posterior (or arthroscopic) approach.
A 44 year old man is involved in a road traffic accident. He suffers significant injuries
to his thorax, he has bilateral haemopneumothoraces and a suspected
haemopericardium. He is to undergo surgery, what is the best method of accessing
these injuries?

Bilateral thoracoscopy and mediastinoscopy A.

Midline sternotomy B.

Bilateral posterolateral thoracotomy C.

Clam shell thoracotomy D.

None of the above E.

Patients with significant mediastinal and lung injuries are best operated on using a
Clam shell thoracotomy. All modes of access involve a degree of compromise. A
sternotomy would give good access to the heart. However, it takes longer to perform
and does not provide good access to the lungs. Trauma should not be managed using
laparoscopy.

Trauma management

The cornerstone of trauma management is embodied in the principles of ATLS.

Following trauma there is a trimodal death distribution:


 Immediately following injury. Typically as result of brain or high spinal
injuries, cardiac or great vessel damage. Salvage rate is low.
 In early hours following injury. In this group deaths are due to phenomena
such as splenic rupture, sub dural haematomas and haemopneumothoraces
 In the days following injury. Usually due to sepsis or multi organ failure.

Aspects of trauma management

 ABCDE approach.
 Tension pneumothoraces will deteriorate with vigorous ventilation attempts.
 External haemorrhage is managed as part of the primary survey. As a rule
tourniquets should not be used. Blind application of clamps will tend to
damage surrounding structures and packing is the preferred method of
haemorrhage control.
 Urinary catheters and naso gastric tubes may need inserting. Be wary of basal
skull fractures and urethral injuries.
 Patients with head and neck trauma should be assumed to have a cervical
spine injury until proven otherwise.

Thoracic injuries

 Simple pneumothorax
 Mediastinal traversing wounds
 Tracheobronchial tree injury
 Haemothorax
 Blunt cardiac injury
 Diaphragmatic injury
 Aortic disruption
 Pulmonary contusion

Management of thoracic trauma

 Simple pneumothorax insert chest drain. Aspiration is risky in trauma as


pneumothorax may be from lung laceration and convert to tension
pneumothorax.
 Mediastinal traversing wounds These result from situations like stabbings.
Exit and entry wounds in separate hemithoraces. The presence of a mediastinal
haematoma indicates the likelihood of a great vessel injury. All patients should
undergo CT angiogram and oesophageal contrast swallow. Indications for
thoracotomy are largely related to blood loss and will be addressed below.
 Tracheobronchial tree injury Unusual injuries. In blunt trauma most injuries
occur within 4cm of the carina. Features suggesting this injury include
haemoptysis and surgical emphysema. These injuries have a very large air leak
and may have tension pneumothorax.
 Haemothorax Usually caused by laceration of lung vessel or internal
mammary artery by rib fracture. Patients should all have a wide bore 36F chest
drain. Indications for thoracotomy include loss of more than 1.5L blood
initially or ongoing losses of >200ml per hour for >2 hours.
 Cardiac contusions Usually cardiac arrhythmias, often overlying sternal
fracture. Perform echocardiography to exclude pericardial effusions and
tamponade. Risk of arrhythmias falls after 24 hours.
 Diaphragmatic injury Usually left sided. Direct surgical repair is performed.
 Traumatic aortic disruption Commonest cause of death after RTA or falls.
Usually incomplete laceration near ligamentum arteriosum. All survivors will
have contained haematoma. Only 1-2% of patients with this injury will have a
normal chest x-ray.
 Pulmonary contusion Common and lethal. Insidious onset. Early intubation
and ventilation.

Abdominal trauma

 Deceleration injuries are common.


 In blunt trauma requiring laparotomy the spleen is most commonly injured
(40%)
 Stab wounds traverse structures most commonly liver (40%)
 Gunshot wounds have variable effects depending upon bullet type. Small
bowel is most commonly injured (50%)
 Patients with stab wounds and no peritoneal signs up to 25% will not enter the
peritoneal cavity
 Blood at urethral meatus suggests a urethral tear
 High riding prostate on PR = urethral disruption
 Mechanical testing for pelvic stability should only be performed once

Investigations in abdominal trauma

USS Abdominal CT scan Diagnostic


Peritoneal Lavage
Document fluid if Document organ injury Document bleeding if Indication
hypotensive if normotensive hypotensive
Early diagnosis, non Most specific for Early diagnosis and Advantages
invasive and localising injury; 92 to sensitive; 98%
repeatable; 86 to 95% 98% accurate accurate
accurate
Operator dependent Location of scanner Invasive and may Disadvantages
and may miss away from facilities, miss retroperitoneal
retroperitoneal injury time taken for reporting, and diaphragmatic
need for contrast injury

 Amylase may be normal following pancreatic trauma


 Urethrography if suspected urethral injury

A 44 year old man is involved in a road traffic accident. He suffers significant injuries
to his thorax, he has bilateral haemopneumothoraces and a suspected
haemopericardium. He is to undergo surgery, what is the best method of accessing
these injuries?

Bilateral thoracoscopy and mediastinoscopy A.

Midline sternotomy B.

Bilateral posterolateral thoracotomy C.

Clam shell thoracotomy D.

None of the above E.

Patients with significant mediastinal and lung injuries are best operated on using a
Clam shell thoracotomy. All modes of access involve a degree of compromise. A
sternotomy would give good access to the heart. However, it takes longer to perform
and does not provide good access to the lungs. Trauma should not be managed using
laparoscopy.

Trauma management

The cornerstone of trauma management is embodied in the principles of ATLS.

Following trauma there is a trimodal death distribution:

 Immediately following injury. Typically as result of brain or high spinal


injuries, cardiac or great vessel damage. Salvage rate is low.
 In early hours following injury. In this group deaths are due to phenomena
such as splenic rupture, sub dural haematomas and haemopneumothoraces
 In the days following injury. Usually due to sepsis or multi organ failure.

Aspects of trauma management

 ABCDE approach.
 Tension pneumothoraces will deteriorate with vigorous ventilation attempts.
 External haemorrhage is managed as part of the primary survey. As a rule
tourniquets should not be used. Blind application of clamps will tend to
damage surrounding structures and packing is the preferred method of
haemorrhage control.
 Urinary catheters and naso gastric tubes may need inserting. Be wary of basal
skull fractures and urethral injuries.
 Patients with head and neck trauma should be assumed to have a cervical
spine injury until proven otherwise.

Thoracic injuries

 Simple pneumothorax
 Mediastinal traversing wounds
 Tracheobronchial tree injury
 Haemothorax
 Blunt cardiac injury
 Diaphragmatic injury
 Aortic disruption
 Pulmonary contusion

Management of thoracic trauma

 Simple pneumothorax insert chest drain. Aspiration is risky in trauma as


pneumothorax may be from lung laceration and convert to tension
pneumothorax.
 Mediastinal traversing wounds These result from situations like stabbings.
Exit and entry wounds in separate hemithoraces. The presence of a mediastinal
haematoma indicates the likelihood of a great vessel injury. All patients should
undergo CT angiogram and oesophageal contrast swallow. Indications for
thoracotomy are largely related to blood loss and will be addressed below.
 Tracheobronchial tree injury Unusual injuries. In blunt trauma most injuries
occur within 4cm of the carina. Features suggesting this injury include
haemoptysis and surgical emphysema. These injuries have a very large air leak
and may have tension pneumothorax.
 Haemothorax Usually caused by laceration of lung vessel or internal
mammary artery by rib fracture. Patients should all have a wide bore 36F chest
drain. Indications for thoracotomy include loss of more than 1.5L blood
initially or ongoing losses of >200ml per hour for >2 hours.
 Cardiac contusions Usually cardiac arrhythmias, often overlying sternal
fracture. Perform echocardiography to exclude pericardial effusions and
tamponade. Risk of arrhythmias falls after 24 hours.
 Diaphragmatic injury Usually left sided. Direct surgical repair is performed.
 Traumatic aortic disruption Commonest cause of death after RTA or falls.
Usually incomplete laceration near ligamentum arteriosum. All survivors will
have contained haematoma. Only 1-2% of patients with this injury will have a
normal chest x-ray.
 Pulmonary contusion Common and lethal. Insidious onset. Early intubation
and ventilation.

Abdominal trauma

 Deceleration injuries are common.


 In blunt trauma requiring laparotomy the spleen is most commonly injured
(40%)
 Stab wounds traverse structures most commonly liver (40%)
 Gunshot wounds have variable effects depending upon bullet type. Small
bowel is most commonly injured (50%)
 Patients with stab wounds and no peritoneal signs up to 25% will not enter the
peritoneal cavity
 Blood at urethral meatus suggests a urethral tear
 High riding prostate on PR = urethral disruption
 Mechanical testing for pelvic stability should only be performed once

Investigations in abdominal trauma

USS Abdominal CT scan Diagnostic


Peritoneal Lavage
Document fluid if Document organ injury Document bleeding if Indication
hypotensive if normotensive hypotensive
Early diagnosis, non Most specific for Early diagnosis and Advantages
invasive and localising injury; 92 to sensitive; 98%
repeatable; 86 to 95% 98% accurate accurate
accurate
Operator dependent Location of scanner Invasive and may Disadvantages
and may miss away from facilities, miss retroperitoneal
retroperitoneal injury time taken for reporting, and diaphragmatic
need for contrast injury

 Amylase may be normal following pancreatic trauma


 Urethrography if suspected urethral injury
 A 63 year old male is admitted to the surgical ward for an elective femoral
popliteal bypass. He suddenly starts complaining of central, crushing chest
pain which is radiating to the left arm. The Nursing staff start high flow
oxygen and give a spray of glyceryl trinitrate spray. Unfortunately there is no
relief of symptoms. What is the next agent to be administered?

Aspirin 75mg A.

Clopidogrel 75mg B.

Aspirin 300mg C.

Clopidogrel 300mg D.

Direct admission to angiography suite E.



Aspirin 300mg should be given as soon as possible. If the patient has a
moderate to high risk of myocardial infarction, then Clopidogrel should be
given with a low molecular weight heparin. Thromboloysis or urgent
percutaneous intervention should be given if there are significant ECG
changes.
 Management of acute coronary syndrome

NICE produced guidelines in 2010 on the management of unstable angina and
non-ST elevation myocardial infarction (NSTEMI). They advocate managing
patients based on the early risk assessment using a recognised scoring system
such as GRACE (Global Registry of Acute Cardiac Events) to calculate a
predicted 6 month mortality.

All patients should receive


- aspirin 300mg
- nitrates or morphine to relieve chest pain if required

Whilst it is common that non-hypoxic patients receive oxygen therapy there is


little evidence to support this approach. The 2008 British Thoracic Society
oxygen therapy guidelines advise not giving oxygen unless the patient is
hypoxic.

{Antithrombin} treatment. Low molecular weight heparin should be offered to


patients who are not at a high risk of bleeding and who are not having
angiography within the next 24 hours. If angiography is likely within 24 hours
or a patients creatinine is > 265 umol/l unfractionated heparin should be given.

{Clopidogrel} 300mg should be given to patients with a predicted 6 month


mortality of more than 1.5% or patients who may undergo percutaneous
coronary intervention within 24 hours of admission to hospital. Clopidogrel
should be continued for 12 months.

Intravenous {glycoprotein IIb/IIIa receptor antagonists} (eptifibatide or


tirofiban) should be given to patients who have an intermediate or higher risk
of adverse cardiovascular events (predicted 6-month mortality above 3.0%),
and who are scheduled to undergo angiography within 96 hours of hospital
admission.

{Coronary angiography} should be considered within 96 hours of first


admission
to hospital to patients who have a predicted 6-month mortality above 3.0%. It
should also be performed as soon as possible in patients who are clinically
unstable.
Which of the following is not a change found on an ECG in acute pulmonary
embolism?

No changes A.

J waves B.

P pulmonale C.

Right ventricular strain D.

T wave inversion in the inferior leads E.

S1, Q3, T3

J waves are pathognomonic of hypothermia.


Pulmonary Embolism: ECG changes

 No changes
 S1, Q3, T3
 Tall R waves: V1
 P pulmonale (peaked P waves): inferior leads
 Right axis deviation, Right bundle branch block
 Atrial arrhythmias
 T wave inversion: V1, V2, V3
 Right ventricular strain: if identified is associated with adverse short-term
outcome and adds prognostic value to echocardiographic evidence of right
ventricular dysfunction in patients with acute pulmonary embolism and
normal blood pressure.

References
Vanni S et al. Prognostic value of ECG among patients with acute pulmonary
embolism and normal blood pressure. Am J Med. 2009 Mar;122(3):257-64.
A 42 year old woman is admitted to surgery with acute cholecystitis. She is known to
have hypertension, rheumatoid arthritis and polymyalgia rheumatica. Her medical
therapy includes:
Paracetamol 1g qds
Codeine phosphate 30mg qds
Bendrofluazide 2.5 mg od
Ramipril 10mg od
Methotrexate 7.5mg once a week
Prednisolone 5mg od
You are called by the Senior House Officer to assess this lady as she has become
delirious and hypotensive 2h after surgery. Her blood results reveal:

132 mmol/l Na+


5.3 mmol/l K+
7 mmol/l Urea
108 µmol/l Creatinine

12.4 g/dl Hb
178 * 109/l Platelets
15.4 * 109/l WBC

What management is needed immediately?

Ceftriaxone IV A.

Hydrocortisone 50mg IV B.

CT scan abdomen C.
Urgent exploration laparotomy D.

Hydrocortisone 100mg IV E.

This patient has acute adrenal insufficiency and urgently needs steroid replacement.

Addisonian crisis

Causes

 Sepsis or surgery causing an acute exacerbation of chronic insufficiency


(Addison's, Hypopituitarism)
 Adrenal haemorrhage eg Waterhouse-Friderichsen syndrome (fulminant
meningococcemia)
 Steroid withdrawal

Management

 Hydrocortisone 100 mg im or iv
 1 litre normal saline infused over 30-60 mins or with dextrose if
hypoglycaemic
 Continue hydrocortisone 6 hourly until the patient is stable. No
fludrocortisone is required because high cortisol exerts weak
mineralocorticoid action
 Oral replacement may begin after 24 hours and be reduced to maintenance
over 3-4 days

Theme: Thoracic trauma

Thoracotomy A.
Manage conservatively B.
Intercostal tube drain insertion C.
CT scanning D.
Bronchoscopy E.
Negative pressure intercostal tube drainage F.
Video assisted thoracoscopy and pleurectomy G.

For each of the following scenarios please select the most appropriate management
option from the list. Each option may be used once, more than once or not at all.

A 28 year old male is involved in a road traffic accident he is thrown from his 15.
motorbike onto the pavement and sustains a haemopneumothorax and flail
segment of the right chest

You answered Negative pressure intercostal tube drainage


The correct answer is Intercostal tube drain insertion

He requires a chest drain and analgesia. In general all haemopneumothoraces


should be managed by intercostal chest drain insertion as they have a risk of
becoming a tension pneumothorax until the lung laceration has sealed.

A 19 year old man is stabbed in the chest at a nightclub. He develops a cardiac 16.
arrest in casualty following an attempted transfer to the CT scanning room

Thoracotomy

This is one indication for an 'emergency room' thoracotomy, there are not
many others! Typical injuries include ventricular penetration, great vessel
disruption and hilar lung injuries.

A 32 year old male falls over and sustains a small pneumothorax following a 17.
simple rib fracture. He has no physiological compromise

You answered Manage conservatively

The correct answer is Intercostal tube drain insertion

Unlike spontaneous pneumothoraces most would advocate chest tube drainage


in the context of pneumothorax resulting from trauma. This is because of the
risk of the lung laceration developing a tension. Once there is no further
evidence of air leak the chest drain may be removed and a check x-ray
performed to check there is no reaccumulation prior to discharge.

Thoracic trauma

Types of thoracic trauma

 Often laceration to lung parenchyma with flap Tension


 Pressure develops in thorax pneumothorax
 Most common cause is mechanical ventilation in patient
with pleural injury
 Symptoms overlap with cardiac tamponade, hyper-
resonant percussion note is more likely in tension
pnemothorax

 Chest wall disconnects from thoracic cage Flail chest


 Multiple rib fractures (at least two fractures per rib in at
least two ribs)
 Associated with pulmonary contusion
 Abnormal chest motion
 Avoid over hydration and fluid overload

 Most common cause is lung laceration with air leakage Pneumothorax


 Most traumatic pneumothoraces should have a chest
drain
 Patients with traumatic pneumothorax should never be
mechanically ventilated until a chest drain is inserted

 Most commonly due to laceration of lung, intercostal Haemothorax


vessel or internal mammary artery
 Haemothoraces large enough to appear on CXR are
treated with large bore chest drain
 Surgical exploration is warranted if >1500ml blood
drained immediately

 Beck's triad: elevated venous pressure, reduced arterial Cardiac tamponade


pressure, reduced heart heart sounds.
 Pulsus paradoxus
 May occur with as little as 100ml blood

 Most common potentially lethal chest injury Pulmonary


 Arterial blood gases and pulse oximetry important contusion
 Early intubation within an hour if significant hypoxia

 Usually occurs secondary to chest wall injury Blunt cardiac injury


 ECG may show features of myocardial infarction
 Sequelae: hypotension, arrhythmias, cardiac wall motion
abnormalities

 Deceleration injuries Aorta disruption


 Contained haematoma
 Widened mediastinum

 Most due to motor vehicle accidents and blunt trauma Diaphragm


causing large radial tears (laceration injuries result in disruption
small tears)
 More common on left side
 Insert gastric tube, which will pass into the thoracic
cavity

 Entrance wound in one haemothorax and exit Mediastinal


wound/foreign body in opposite haemothorax traversing wounds
 Mediastinal haematoma or pleural cap suggests great
vessel injury
 Mortality is 20%
References
ATLS Manual 8th Edition
A 21 year old man is undergoing an inguinal hernia repair and receives a does of
intravenous co-amoxyclav. He is reported to have a penicillin allergy. Over the next
few minutes his vital signs are: Pulse - 130bpm, blood pressure- 60/40mmHg. What is
the first line treatment?

Hydrocortisone 100mg IV A.

Adrenaline 1:1000 IV B.

Chlorpheniramine 10mg IV C.

Adrenaline 1:1000 IM D.

Adrenaline 1:10000 IV E.

Theme from 2009 Exam


The first line treatment of anaphylactic shock is intra muscular adrenaline.

Anaphylactic shock

 Suspect if there has been exposure to an allergen

Management

- Remove allergen
- ABCD
- Drugs:

Adrenaline 1:1000 0.5ml INTRAMUSCULARLY (not IV). Repeat after 5 mins if no


response.
Then Chlorpheniramine 10mg IV
Then Hydrocortisone 100-200mg IV

Reference
Emergency treatment of anaphylactic reactions. Guidelines for healthcare providers.
Working Group of the Resuscitation Council (UK).2008
heme: Types of stroke

Anterior cerebral artery infarct A.


Middle cerebral artery infarct B.
Posterior cerebral artery infarct C.
Pituitary mass D.
Lacunar infarct E.
Lateral medullary syndrome F.
Pontine infarct G.
Horner's syndrome H.
Cerebellar infarct I.

Please select the most likely cause for the symptoms given. Each option may be used
once, more than once or not at all.

A 53 year old teacher is admitted to the vascular ward for a carotid 19.
endarterectomy. Your houseman does a preoperative assessment and notes that
there is a right homonymous hemianopia. There is no other neurology.

Posterior cerebral artery infarct

This patient has had a left occipital infarct, as there is only a homonymous
hemianopia. If this patient had a temporal or parietal lobe infarct, there would
be associated hemiparesis and higher cortical dysfunction. This is important to
differentiate, as the carotid endarterectomy is inappropriate in this patient as
the lesion is in the posterior cerebral artery.

A 52 year man is admitted to the vascular ward for an amputation. The patient 20.
complains of unsteadiness. On further examination you detect right facial
numbness and right sided nystagmus. There is sensory loss of the left side.

You answered Pontine infarct

The correct answer is Lateral medullary syndrome

A combination of ipsilateral ataxia, nystagmus, dysphagia, facial numbness,


cranial nerve palsy with contralateral hemisensory loss indicates this diagnosis.

A 48 year old type 2 diabetic complains of numbness in his left arm and leg. 21.
Otherwise there is no other neurological signs.

Lacunar infarct

Isolated hemisensory loss is a feature of a lacunar infarct.

Stroke: types

 Presents with headache, vomiting, loss of Primary intracerebral


haemorrhage (PICH, c.
consciousness 10%)

 Involves middle and anterior cerebral arteries Total anterior circulation


 Hemiparesis/hemisensory loss infarcts (TACI, c. 15%)
 Homonymous hemianopia
 Higher cognitive dysfunction e.g. Dysphasia

 Involves smaller arteries of anterior circulation Partial anterior circulation


e.g. upper or lower division of middle cerebral infarcts (PACI, c. 25%)
artery
 Higher cognitive dysfunction or two of the three
TACI features

 Involves perforating arteries around the internal Lacunar infarcts (LACI, c.


capsule, thalamus and basal ganglia 25%)
 Present with either isolated hemiparesis,
hemisensory loss or hemiparesis with limb ataxia

 Vertebrobasilar arteries Posterior circulation


 Presents with features of brainstem damage infarcts (POCI, c. 25%)
 Ataxia, disorders of gaze and vision, cranial
nerve lesions

 Wallenberg's syndrome Lateral medullary


 Ipsilateral: ataxia, nystagmus, dysphagia, facial syndrome (posterior
numbness, cranial nerve palsy e.g. Horner's inferior cerebellar artery)
 Contralateral: limb sensory loss

 Ipsilateral III palsy Weber's syndrome


 Contralateral weakness

--------------------------------------

Anterior cerebral artery

 Contralateral hemiparesis and sensory loss, lower extremity > upper


 Disconnection syndrome

Middle cerebral artery

 Contralateral hemiparesis and sensory loss, upper extremity > lower


 Contralateral hemianopia
 Aphasia (Wernicke's)
 Gaze abnormalities
Posterior cerebral artery

 Contralateral hemianopia with macular sparing


 Disconnection syndrome

Lacunar

 Present with either isolated hemiparesis, hemisensory loss or hemiparesis with


limb ataxia

Lateral medulla (posterior inferior cerebellar artery)

 Ipsilateral: ataxia, nystagmus, dysphagia, facial numbness, cranial nerve palsy


e.g.

Horner's

 Contralateral: limb sensory loss

Pontine

 VI nerve: horizontal gaze palsy


 VII nerve
 Contralateral hemiparesis

A 22 year old man has a full thickness burn on his chest. It is well circumscribed. In
A&E his saturations are reduced to 92% on 15L Oxygen, Blood pressure 102/66
mmHg and HR 105bpm. What is the best management?

Haemodialysis A.

Escharotomy B.

Fasciotomy C.

Cardiac bypass D.

Non invasive ventilation E.

The chest burn and its associated oedema is limiting respiration. Therefore an
escharotomy of the chest is indicated, allowing the breast plate to mobilise causing
ventilation.
Burns

Types of burn

Management Blanching Skin Skin layers Type of burn


appearance affected
Yes Red, moist Epidermis Epidermal/Superficial
Normally heals Yes Pale, dry Epidermis and Superficial partial
with no part of papillary thickness
intervention dermis affected
Needs surgical No Mottled red Epidermis, whole Deep partial thickness
intervention colour papillary dermis
(depending on affected
site)
Burns centre No Dry, leathery Whole skin layer Full thickness
hard wound and subcutaneous
tissue affected

Depth of burn assessment

 Bleeding on needle prick


 Sensation
 Appearance
 Blanching to pressure

Percentage burn estimation


Lund Browder chart: most accurate even in children
Wallace rule of nines
Palmar surface: surface area palm = 0.8% burn

>15% body surface area burns in adults needs urgent burn fluid resuscitation

Transfer to burn centre if:

 Need burn shock resuscitation


 Face/hands/genitals affected
 Deep partial thickness or full thickness burns
 Significant electrical/chemical burns

Escharotomies

 Indicated in circumferential full thickness burns to the torso or limbs.


 Careful division of the encasing band of burn tissue will potentially improve
ventilation (if the burn involves the torso), or relieve compartment syndrome
and oedema (where a limb is involved)

References
www.euroburn.org/e107files/downloads/guidelinesburncare.pdf

Hettiaratchy S & Papini R. Initial management of a major burn: assessment and


resuscitation. BMJ 2004;329:101-103
A 16 year old man sustains a basal skull fracture and is suspected of having CSF
rhinorrhoea. Which of the following laboratory tests would most accurately identify
whether CSF is present or not?

Microscopy to identify red blood cells A.

Lab stix testing for glucose B.

Lab stix testing for protein C.

Beta 2 transferrin assay D.

Microscopy, gram stain and culture E.

Beta 2 transferrin is a carbohydrate free form of transferrin that is almost exclusively


found in the CSF. Although lab stix testing for glucose is traditional it is associated
with false positive results secondary to contamination with other glucose containing
bodily secretions.

Head injury management- NICE Guidelines

Summary of guidelines

 All patients should be assessed within 15 minutes on arrival to A&E


 Document all 3 components of the GCS
 If GCS <8 or = to 8, consider stabilising the airway
 Treat pain with low dose IV opiates (if safe)
 Full spine immobilisation until assessment if:

- GCS < 15
- neck pain/tenderness
- paraesthesia extremities
- focal neurological deficit
- suspected c-spine injury

If a c-spine injury is suspected a 3 view c-spine x-ray is indicated. CT c-spine is


preferred if:
- Intubated
- GCS <13
- Normal x-ray but continued concerns regarding c-spine injury

Immediate CT head (within 1h) if:

 GCS < 13 on admission


 GCS < 15 2h after admission
 Suspected open or depressed skull fracture
 Suspected skull base fracture (panda eyes, Battle's sign, CSF from nose/ear,
bleeding ear)
 Focal neurology
 Vomiting > 1 episode
 Post traumatic seizure
 Coagulopathy

Contact neurosurgeon if:

 Persistent GCS < 8 or = 8


 Unexplained confusion > 4h
 Reduced GCS after admission
 Progressive neurological signs
 Incomplete recovery post seizure
 Penetrating injury
 Cerebrospinal leak

Observations

 1/2 hourly GCS until 15

Reference
http://guidance.nice.org.uk/CG56/QuickRefGuide/pdf/English
A 66 year old male is admitted to the vascular ward for an amputation. He reports
episodes of vertigo and dysarthria to the house officer. He sudden;y collapses with a
glasgow coma score of 3. What is the most likely diagnosis?

Cerebral haemorrhage in left temporal parietal area A.

Opiate overdose B.

Cerebral haemorrhage in right temporal parietal area C.

Diazepam overdose D.

Basilar artery occlusion E.

Vertigo and dysarthria suggest a posterior circulation event. In the scenario of a


patient complaining of posterior symptoms and a sudden deterioration in
consciousness, the main differential diagnosis is of a basilar artery occlusion.

Stroke: types
 Presents with headache, vomiting, loss of Primary intracerebral
consciousness haemorrhage (PICH, c.
10%)
 Involves middle and anterior cerebral arteries Total anterior circulation
 Hemiparesis/hemisensory loss infarcts (TACI, c. 15%)
 Homonymous hemianopia
 Higher cognitive dysfunction e.g. Dysphasia

 Involves smaller arteries of anterior circulation Partial anterior circulation


e.g. upper or lower division of middle cerebral infarcts (PACI, c. 25%)
artery
 Higher cognitive dysfunction or two of the three
TACI features

 Involves perforating arteries around the internal Lacunar infarcts (LACI, c.


capsule, thalamus and basal ganglia 25%)
 Present with either isolated hemiparesis,
hemisensory loss or hemiparesis with limb ataxia

 Vertebrobasilar arteries Posterior circulation


 Presents with features of brainstem damage infarcts (POCI, c. 25%)
 Ataxia, disorders of gaze and vision, cranial
nerve lesions

 Wallenberg's syndrome Lateral medullary


 Ipsilateral: ataxia, nystagmus, dysphagia, facial syndrome (posterior
numbness, cranial nerve palsy e.g. Horner's inferior cerebellar artery)
 Contralateral: limb sensory loss

 Ipsilateral III palsy Weber's syndrome


 Contralateral weakness

--------------------------------------

Anterior cerebral artery

 Contralateral hemiparesis and sensory loss, lower extremity > upper


 Disconnection syndrome

Middle cerebral artery

 Contralateral hemiparesis and sensory loss, upper extremity > lower


 Contralateral hemianopia
 Aphasia (Wernicke's)
 Gaze abnormalities
Posterior cerebral artery

 Contralateral hemianopia with macular sparing


 Disconnection syndrome

Lacunar

 Present with either isolated hemiparesis, hemisensory loss or hemiparesis with


limb ataxia

Lateral medulla (posterior inferior cerebellar artery)

 Ipsilateral: ataxia, nystagmus, dysphagia, facial numbness, cranial nerve palsy


e.g.

Horner's

 Contralateral: limb sensory loss

Pontine

 VI nerve: horizontal gaze palsy


 VII nerve
 Contralateral hemiparesis

A 19 year intravenous drug abuser is recovering following a surgical drainage of a


psoas abscess. He is found collapsed in the ward toilet unresponsive and with pinpoint
pupils. What is the most appropriate immediate management?

Intravenous flumazenil A.

Intravenous nalaxone B.

Intravenous benxhexol C.

No further management D.

Intravenous glycopyrolate E.

Intravenous nalaxone is needed to treat the patient who has had an overdose of opiate.
Naloxone has the quickest onset of action, however it is important to be aware of its
short acting duration and the need for further administration. There is also the risk of
rebound pain once naloxone is given.
Opioid misuse

Opioids are substances which bind to opioid receptors. This includes both naturally
occurring opiates such as morphine and synthetic opioids such as buprenorphine and
methadone.

Features of opioid misuse

 Rhinorrhoea
 Needle track marks
 Pinpoint pupils
 Drowsiness

Complications of intravenous opioid misuse

 Viral infection secondary to sharing needles: HIV, hepatitis B & C


 Bacterial infection secondary to injection: infective endocarditis, septic
arthritis, septicaemia, necrotising fasciitis, groin abscess
 Pseudoaneurysm
 Venous thromboembolism
 Osteomyelitis
 Overdose may lead to respiratory depression and death

Emergency management of opioid overdose

 IV or IM naloxone: has a rapid onset and relatively short duration of action

A 68 year old male is admitted to the surgical ward for assessment of severe
epigastric pain. His abdomen is soft and non tender. However the Nurse forces you to
look at the ECG. It looks abnormal. Which of the following features is an indication
for urgent coronary thrombolysis or percutaneous intervention?

Right bundle branch block A.

ST elevation of 1mm in leads V1 to V6 B.

Ventricular tachycardia C.

Q waves in leads V1 to V6 D.

ST elevation of 1mm in leads II, III and aVF E.


ECG changes for thrombolysis or percutaneous intervention:
ST elevation of > 2mm (2 small squares) in 2 or more consecutive anterior leads (V1-
V6) OR

ST elevation of greater than 1mm (1 small square) in greater than 2 consecutive


inferior leads (II, III, avF, avL) OR

New Left bundle branch block

ST elevation of 1mm in leads II, III and aVF reflects significant cardiac ischaemia
due to the right coronary artery occlusion. The medical registrar should be contacted
to urgently assess the patient. Note right coronary artery occlusions puts the patient at
risk of cardiac arrhythmias (due to blood supply to the sino atrial node).

Thrombolysis or percutaneous intervention in myocardial infarction

Thrombolytic drugs activate plasminogen to form plasmin. This in turn degrades


fibrin and help breaks up thrombi. They in primarily used in patients who present with
a ST elevation myocardial infarction. Other indications include acute ischaemic stroke
and pulmonary embolism, although strict inclusion criteria apply.

Examples

 alteplase
 tenecteplase
 streptokinase

Contraindications to thrombolysis

 active internal bleeding


 recent haemorrhage, trauma or surgery (including dental extraction)
 coagulation and bleeding disorders
 intracranial neoplasm
 stroke < 3 months
 aortic dissection
 recent head injury
 pregnancy
 severe hypertension

Side-effects

 haemorrhage
 hypotension - more common with streptokinase
 allergic reactions may occur with streptokinase

Which of the following is not typically associated with a degloving injury?

Overlying pallor of the skin A.

Abnormal motility of the overlying skin B.

History of friction type injury C.


Improved results when the degloved segment is left in situ as a D.
temporary closure

Poor results when primary compression treatment is used in E.


preference to skin grafting

Degloving injuries typically involve extremities and are usually friction injuries eg
arm being run over. There is abnormal motility of the overlying skin, pallor, loss of
sensation. Early treatment is key and should involve skin grafting which may use the
degloved segment. This however, should be formally prepared for the role and simple
compression bandaging gives poor results.

Wound healing

Surgical wounds are either incisional or excisional and either clean, clean
contaminated or dirty. Although the stages of wound healing are broadly similar their
contributions will vary according to the wound type.

The main stages of wound healing include:

Haemostasis

 Vasospasm in adjacent vessels, platelet plug formation and generation of


fibrin rich clot.

Inflammation

 Neutrophils migrate into wound (function impaired in diabetes).


 Growth factors released, including basic fibroblast growth factor and vascular
endothelial growth factor.
 Fibroblasts replicate within the adjacent matrix and migrate into wound.
 Macrophages and fibroblasts couple matrix regeneration and clot substitution.

Regeneration

 Platelet derived growth factor and transformation growth factors stimulate


fibroblasts and epithelial cells.
 Fibroblasts produce a collagen network.
 Angiogenesis occurs and wound resembles granulation tissue.

Remodeling

 Longest phase of the healing process and may last up to one year (or longer).
 During this phase fibroblasts become differentiated (myofibroblasts) and these
facilitate wound contraction.
 Collagen fibres are remodeled.
 Microvessels regress leaving a pale scar.

The above description represents an idealised scenario. A number of diseases may


distort this process. It is obvious that one of the key events is the establishing well
vascularised tissue. At a local level angiogenesis occurs, but if arterial inflow and
venous return are compromised then healing may be impaired, or simply nor occur at
all. The results of vascular compromise are all too evidence in those with peripheral
vascular disease or those poorly constructed bowel anastomoses.

Conditions such as jaundice will impair fibroblast synthetic function and overall
immunity with a detrimental effect in most parts of healing.

Problems with scars:

Hypertrophic scars
Excessive amounts of collagen within a scar. Nodules may be present histologically
containing randomly arranged fibrils within and parallel fibres on the surface. The
tissue itself is confined to the extent of the wound itself and is usually the result of a
full thickness dermal injury. They may go on to develop contractures.

Image of hypertrophic scarring. Note that it remains confined to the boundaries of the
original wound:

Image sourced from Wikipedia

Keloid scars
Excessive amounts of collagen within a scar. Typically a keloid scar will pass beyond
the boundaries of the original injury. They do not contain nodules and may occur
following even trivial injury. They do not regress over time and may recur following
removal.

Image of a keloid scar. Note the extension beyond the boundaries of the original
incision:
Image sourced from Wikipedia

Drugs which impair wound healing:

 Non steroidal anti inflammatory drugs


 Steroids
 Immunosupressive agents
 Anti neoplastic drugs

Closure
Delayed primary closure is the anatomically precise closure that is delayed for a few
days but before granulation tissue becomes macroscopically evident.

Secondary closure refers to either spontaneous closure or to surgical closure after


granulation tissue has formed.
Which of the following statements relating to large volume blood loss in trauma is
incorrect?

Tranexamic acid reduces the incidence of rebleeding following A.


surgery

Hypocalcaemia may complicate resuscitation B.

Colloids are preferred initially as they reduce the incidence of C.


coagulopathy

When patients receive over 5 units of whole blood mortality increases D.


when blood products greater than 3 weeks old are utilised

In the battlefield setting a ratio of 1 unit fresh blood to 1 unit plasma E.


is often utilised
Fresh blood is the fluid of choice when large volume blood loss complicates trauma.
Mortality is doubled when blood >3 weeks old is used.

Trauma management

The cornerstone of trauma management is embodied in the principles of ATLS.

Following trauma there is a trimodal death distribution:

 Immediately following injury. Typically as result of brain or high spinal


injuries, cardiac or great vessel damage. Salvage rate is low.
 In early hours following injury. In this group deaths are due to phenomena
such as splenic rupture, sub dural haematomas and haemopneumothoraces
 In the days following injury. Usually due to sepsis or multi organ failure.

Aspects of trauma management

 ABCDE approach.
 Tension pneumothoraces will deteriorate with vigorous ventilation attempts.
 External haemorrhage is managed as part of the primary survey. As a rule
tourniquets should not be used. Blind application of clamps will tend to
damage surrounding structures and packing is the preferred method of
haemorrhage control.
 Urinary catheters and naso gastric tubes may need inserting. Be wary of basal
skull fractures and urethral injuries.
 Patients with head and neck trauma should be assumed to have a cervical
spine injury until proven otherwise.

Thoracic injuries

 Simple pneumothorax
 Mediastinal traversing wounds
 Tracheobronchial tree injury
 Haemothorax
 Blunt cardiac injury
 Diaphragmatic injury
 Aortic disruption
 Pulmonary contusion

Management of thoracic trauma

 Simple pneumothorax insert chest drain. Aspiration is risky in trauma as


pneumothorax may be from lung laceration and convert to tension
pneumothorax.
 Mediastinal traversing wounds These result from situations like stabbings.
Exit and entry wounds in separate hemithoraces. The presence of a mediastinal
haematoma indicates the likelihood of a great vessel injury. All patients should
undergo CT angiogram and oesophageal contrast swallow. Indications for
thoracotomy are largely related to blood loss and will be addressed below.
 Tracheobronchial tree injury Unusual injuries. In blunt trauma most injuries
occur within 4cm of the carina. Features suggesting this injury include
haemoptysis and surgical emphysema. These injuries have a very large air leak
and may have tension pneumothorax.
 Haemothorax Usually caused by laceration of lung vessel or internal
mammary artery by rib fracture. Patients should all have a wide bore 36F chest
drain. Indications for thoracotomy include loss of more than 1.5L blood
initially or ongoing losses of >200ml per hour for >2 hours.
 Cardiac contusions Usually cardiac arrhythmias, often overlying sternal
fracture. Perform echocardiography to exclude pericardial effusions and
tamponade. Risk of arrhythmias falls after 24 hours.
 Diaphragmatic injury Usually left sided. Direct surgical repair is performed.
 Traumatic aortic disruption Commonest cause of death after RTA or falls.
Usually incomplete laceration near ligamentum arteriosum. All survivors will
have contained haematoma. Only 1-2% of patients with this injury will have a
normal chest x-ray.
 Pulmonary contusion Common and lethal. Insidious onset. Early intubation
and ventilation.

Abdominal trauma

 Deceleration injuries are common.


 In blunt trauma requiring laparotomy the spleen is most commonly injured
(40%)
 Stab wounds traverse structures most commonly liver (40%)
 Gunshot wounds have variable effects depending upon bullet type. Small
bowel is most commonly injured (50%)
 Patients with stab wounds and no peritoneal signs up to 25% will not enter the
peritoneal cavity
 Blood at urethral meatus suggests a urethral tear
 High riding prostate on PR = urethral disruption
 Mechanical testing for pelvic stability should only be performed once

Investigations in abdominal trauma

USS Abdominal CT scan Diagnostic


Peritoneal Lavage
Document fluid if Document organ injury Document bleeding if Indication
hypotensive if normotensive hypotensive
Early diagnosis, non Most specific for Early diagnosis and Advantages
invasive and localising injury; 92 to sensitive; 98%
repeatable; 86 to 95% 98% accurate accurate
accurate
Operator dependent Location of scanner Invasive and may Disadvantages
and may miss away from facilities, miss retroperitoneal
retroperitoneal injury time taken for reporting, and diaphragmatic
need for contrast injury

 Amylase may be normal following pancreatic trauma


 Urethrography if suspected urethral injury

The following features are typical of superficial dermal burns except:

They are usually erythematous A.

Blistering should not occur B.

Spontaneous healing will occur in nearly all cases C.

They do not extend deeper than the proximal dermal papillae D.

Capillary return should be present E.

Superficial dermal burns are typically erythematous, do not extend beyond the upper
part of the dermal papillae, capillary return and blisters are both usually present.

A Typical example of a first degree burn is shown below

Image sourced from Wikipedia

Burns

Types of burn

Management Blanching Skin Skin layers Type of burn


appearance affected
Yes Red, moist Epidermis Epidermal/Superficial
Normally heals Yes Pale, dry Epidermis and Superficial partial
with no part of papillary thickness
intervention dermis affected
Needs surgical No Mottled red Epidermis, whole Deep partial thickness
intervention colour papillary dermis
(depending on affected
site)
Burns centre No Dry, leathery Whole skin layer Full thickness
hard wound and subcutaneous
tissue affected

Depth of burn assessment

 Bleeding on needle prick


 Sensation
 Appearance
 Blanching to pressure

Percentage burn estimation


Lund Browder chart: most accurate even in children
Wallace rule of nines
Palmar surface: surface area palm = 0.8% burn

>15% body surface area burns in adults needs urgent burn fluid resuscitation

Transfer to burn centre if:

 Need burn shock resuscitation


 Face/hands/genitals affected
 Deep partial thickness or full thickness burns
 Significant electrical/chemical burns

Escharotomies

 Indicated in circumferential full thickness burns to the torso or limbs.


 Careful division of the encasing band of burn tissue will potentially improve
ventilation (if the burn involves the torso), or relieve compartment syndrome
and oedema (where a limb is involved)

References
www.euroburn.org/e107files/downloads/guidelinesburncare.pdf

Hettiaratchy S & Papini R. Initial management of a major burn: assessment and


resuscitation. BMJ 2004;329:101-103
Which of the following is not a feature found on a CXR in traumatic aortic
disruption?

Widened mediastinum A.

Trachea deviated to the left B.

Depression of the left main stem bronchus C.

Obliteration of the aortic knob D.

Widened paraspinal interfaces E.

The trachea is normally deviated to the right.

Thoracic aorta rupture

 Mechanism of injury: Decelerating force i.e. RTA, fall from a great height
 Most people die at scene
 Survivors may have an incomplete laceration at the ligamentum arteriosum of
the aorta.

Clinical features

 Contained haematoma: persistent hypotension


 Detected mainly by history, CXR changes

CXR changes

 Widened mediastinum
 Trachea/Oesophagus to right
 Depression of left main stem bronchus
 Widened paratracheal stripe/paraspinal interfaces
 Space between aorta and pulmonary artery obliterated
 Rib fracture/left haemothorax

Diagnosis
Angiography, usually CT aortogram.

Treatment
Repair or replacement. Ideally they should undergo endovascular repair.
Theme: Head injury management

Observation A.
CT head within 1h B.
CT head within 8h C.
Urgent neurosurgical review (even before CT head performed) D.
3 view c-spine xray E.
2 view c-spine xray F.
CT c-spine G.
MRI c-spine H.

What is the best initial management plan for the scenario given? Each option may be
used once, more than once or not at all.

A 22 year old male falls of a ladder. He complains of neck pain and cannot feel 31.
his legs. His GCS suddenly deteriorates and a CT head confirms an extradural
haematoma. What is the best imaging for his neck?

You answered MRI c-spine

The correct answer is CT c-spine

This man needs a CT scan of his c-spine. A CT scan will give the best
resolution of any bony injury.

A 25 year old teacher falls down the stairs. She complains of a headache and 32.
has vomited 3 times. She has a GCS of 15/15.

CT head within 1h

This lady has a head injury and vomiting > 1, therefore an urgent CT head is
indicated.

An 18 year old student is shot in the back of the head. 33.

You answered CT head within 1h

The correct answer is Urgent neurosurgical review (even before CT head


performed)

A penetrating injury needs urgent neurosurgical review.

Head injury management- NICE Guidelines

Summary of guidelines
 All patients should be assessed within 15 minutes on arrival to A&E
 Document all 3 components of the GCS
 If GCS <8 or = to 8, consider stabilising the airway
 Treat pain with low dose IV opiates (if safe)
 Full spine immobilisation until assessment if:

- GCS < 15
- neck pain/tenderness
- paraesthesia extremities
- focal neurological deficit
- suspected c-spine injury

If a c-spine injury is suspected a 3 view c-spine x-ray is indicated. CT c-spine is


preferred if:
- Intubated
- GCS <13
- Normal x-ray but continued concerns regarding c-spine injury

Immediate CT head (within 1h) if:

 GCS < 13 on admission


 GCS < 15 2h after admission
 Suspected open or depressed skull fracture
 Suspected skull base fracture (panda eyes, Battle's sign, CSF from nose/ear,
bleeding ear)
 Focal neurology
 Vomiting > 1 episode
 Post traumatic seizure
 Coagulopathy

Contact neurosurgeon if:

 Persistent GCS < 8 or = 8


 Unexplained confusion > 4h
 Reduced GCS after admission
 Progressive neurological signs
 Incomplete recovery post seizure
 Penetrating injury
 Cerebrospinal leak

Observations

 1/2 hourly GCS until 15

Reference
http://guidance.nice.org.uk/CG56/QuickRefGuide/pdf/English
Theme: Head injury management

Observation A.
CT head within 1h B.
CT head within 8h C.
Urgent neurosurgical review (even before CT head performed) D.
3 view c-spine xray E.
2 view c-spine xray F.
CT c-spine G.
MRI c-spine H.

What is the best initial management plan for the scenario given? Each option may be
used once, more than once or not at all.

A 22 year old male falls of a ladder. He complains of neck pain and cannot feel 31.
his legs. His GCS suddenly deteriorates and a CT head confirms an extradural
haematoma. What is the best imaging for his neck?

You answered MRI c-spine

The correct answer is CT c-spine

This man needs a CT scan of his c-spine. A CT scan will give the best
resolution of any bony injury.

A 25 year old teacher falls down the stairs. She complains of a headache and 32.
has vomited 3 times. She has a GCS of 15/15.

CT head within 1h

This lady has a head injury and vomiting > 1, therefore an urgent CT head is
indicated.

An 18 year old student is shot in the back of the head. 33.

You answered CT head within 1h

The correct answer is Urgent neurosurgical review (even before CT head


performed)

A penetrating injury needs urgent neurosurgical review.

Head injury management- NICE Guidelines


Summary of guidelines

 All patients should be assessed within 15 minutes on arrival to A&E


 Document all 3 components of the GCS
 If GCS <8 or = to 8, consider stabilising the airway
 Treat pain with low dose IV opiates (if safe)
 Full spine immobilisation until assessment if:

- GCS < 15
- neck pain/tenderness
- paraesthesia extremities
- focal neurological deficit
- suspected c-spine injury

If a c-spine injury is suspected a 3 view c-spine x-ray is indicated. CT c-spine is


preferred if:
- Intubated
- GCS <13
- Normal x-ray but continued concerns regarding c-spine injury

Immediate CT head (within 1h) if:

 GCS < 13 on admission


 GCS < 15 2h after admission
 Suspected open or depressed skull fracture
 Suspected skull base fracture (panda eyes, Battle's sign, CSF from nose/ear,
bleeding ear)
 Focal neurology
 Vomiting > 1 episode
 Post traumatic seizure
 Coagulopathy

Contact neurosurgeon if:

 Persistent GCS < 8 or = 8


 Unexplained confusion > 4h
 Reduced GCS after admission
 Progressive neurological signs
 Incomplete recovery post seizure
 Penetrating injury
 Cerebrospinal leak

Observations

 1/2 hourly GCS until 15


Reference
http://guidance.nice.org.uk/CG56/QuickRefGuide/pdf/English
A 60-year-old man develops palpitations while on the acute surgical unit. An ECG
shows a broad complex tachycardia at a rate of 150 bpm. His blood pressure is 124/82
mmHg and there is no evidence of heart failure. The surgical consultant wants to give
rate control (the medical team are not answering their bleeps). Which one of the
following is it least appropriate to give?

Procainamide A.

Lidocaine B.

Synchronised DC shock C.

Adenosine D.

Verapamil E.
Ventricular tachycardia -
verapamil is contraindicated

Verapamil should never be given to a patient with a broad complex tachycardia as it


may precipitate ventricular fibrillation in patients with ventricular tachycardia.
Adenosine is sometimes given in this situation as a 'trial' if there is a strong suspicion
the underlying rhythm is a supraventricular tachycardia with aberrant conduction

Ventricular tachycardia: management

Whilst a broad complex tachycardia may result from a supraventricular rhythm with
aberrant conduction, the European Resuscitation Council advise that in a peri-arrest
situation it is assumed to be ventricular in origin

If the patient has adverse signs (systolic BP < 90 mmHg, chest pain, heart failure or
rate > 150 beats/min) then immediate cardioversion is indicated. In the absence of
such signs antiarrhythmics may be used. If these fail, then electrical cardioversion
may be needed with synchronised DC shocks

Drug therapy

 amiodarone: ideally administered through a central line


 lidocaine: use with caution in severe left ventricular impairment
 procainamide

Verapamil should NOT be used in VT

If drug therapy fails


 electrophysiological study (EPS)
 implant able cardioverter-defibrillator (ICD) - this is particularly indicated in
patients with significantly impaired LV function

A 24 year old man is admitted to A&E with 35% full thickness burns after being
involved in a house fire. Which fluid is normally avoided during resuscitation in the
first 8-12h?

Plasmalyte A.

Hartmann's B.

Albumin solution C.

Dextrose saline D.

Dextran 40 E.

Albumin causes increased fluid into the interstitial space, therefore is avoided in the
first 8-24h (variable between different departments).

Fluid resuscitation burns

Indication: >15% total body area burns in adults (>10% children)

 The main aim of resuscitation is to prevent the burn deepening


 Most fluid is lost 24h after injury
 First 8-12h fluid shifts from intravascular to interstitial fluid compartments
 Therefore circulatory volume can be compromised. However fluid
resuscitation causes more fluid into the interstitial compartment especially
colloid (therefore avoided in first 8-24h)
 Protein loss occurs

Fluid resuscitation formula


Parkland formula
(Crystalloid only e.g. Hartman's solution/Ringers' lactate)
Total fluid requirement in 24 hours =
4 ml x (total burn surface area (%)) x (body weight (kg))

 50% given in first 8 hours


 50% given in next 16 hours

Resuscitation endpoint:Urine output of 0.5-1.0 ml/kg/hour in adults (increase rate of


fluid to achieve this)
Points to note:

 Starting point of resuscitation is time of injury


 Deduct fluids already given

After 24 hours

 Colloid infusion is begun at a rate of 0.5 ml x(total burn surface area


(%))x(body weight (kg))
 Maintenance crystalloid (usually dextrose-saline) is continued at a rate of 1.5
ml x(burn area)x(body weight)
 Colloids used include albumin and FFP
 Antioxidants, such as vitamin C, can be used to minimize oxidant-mediated
contributions to the inflammatory cascade in burns
 High tension electrical injuries and inhalation injuries require more fluid
 Monitor: packed cell volume, plasma sodium, base excess, and lactate

A 62 year old male attends the hernia clinic. He suddenly develops speech problems,
left facial weakness and left sided arm and leg weakness lasting longer than 5
minutes. What is the next line of management?

Aspirin 300mg A.

Aspirin 75 mg B.

Clopidogrel 300mg C.

Urgent referral for thrombolysis D.

Carotid endarterectomy E.

This patient is within 3h of symptom onset of a stroke. Therefore he should be


urgently referred to the medical team for thrombolysis, before Aspirin is given. This
is an example of the type of medical problem you should be aware of as a surgeon, as
ultimately you can make a difference by referring QUICKLY to the correct specialty
for management.

Stroke: types

 Presents with headache, vomiting, loss of Primary intracerebral


consciousness haemorrhage (PICH, c.
10%)
 Involves middle and anterior cerebral arteries Total anterior circulation
 Hemiparesis/hemisensory loss infarcts (TACI, c. 15%)
 Homonymous hemianopia
 Higher cognitive dysfunction e.g. Dysphasia

 Involves smaller arteries of anterior circulation Partial anterior circulation


e.g. upper or lower division of middle cerebral infarcts (PACI, c. 25%)
artery
 Higher cognitive dysfunction or two of the three
TACI features

 Involves perforating arteries around the internal Lacunar infarcts (LACI, c.


capsule, thalamus and basal ganglia 25%)
 Present with either isolated hemiparesis,
hemisensory loss or hemiparesis with limb ataxia

 Vertebrobasilar arteries Posterior circulation


 Presents with features of brainstem damage infarcts (POCI, c. 25%)
 Ataxia, disorders of gaze and vision, cranial
nerve lesions

 Wallenberg's syndrome Lateral medullary


 Ipsilateral: ataxia, nystagmus, dysphagia, facial syndrome (posterior
numbness, cranial nerve palsy e.g. Horner's inferior cerebellar artery)
 Contralateral: limb sensory loss

 Ipsilateral III palsy Weber's syndrome


 Contralateral weakness

--------------------------------------

Anterior cerebral artery

 Contralateral hemiparesis and sensory loss, lower extremity > upper


 Disconnection syndrome

Middle cerebral artery

 Contralateral hemiparesis and sensory loss, upper extremity > lower


 Contralateral hemianopia
 Aphasia (Wernicke's)
 Gaze abnormalities

Posterior cerebral artery

 Contralateral hemianopia with macular sparing


 Disconnection syndrome

Lacunar
 Present with either isolated hemiparesis, hemisensory loss or hemiparesis with
limb ataxia

Lateral medulla (posterior inferior cerebellar artery)

 Ipsilateral: ataxia, nystagmus, dysphagia, facial numbness, cranial nerve palsy


e.g.

Horner's

 Contralateral: limb sensory loss

Pontine

 VI nerve: horizontal gaze palsy


 VII nerve
 Contralateral hemiparesis

A 45-year-old man is seen in the Emergency Department with nausea, pallor and
lethargy. He has no past medical history of note. A cannula is inserted and serum urea
and electrolytes show the following

140 mmol/l Na+


6.7 mmol/l K+
14 mmol/l Bicarbonate
18.2 mmol/l Urea
230 µmol/l Creatinine

What is the most appropriate initial management?

Nebulised salbutamol A.

Intravenous bicarbonate B.

Haemodialysis C.

Insulin/dextrose infusion D.

Intravenous calcium gluconate E.

The first priority in this patient is to stabilise the myocardium with intravenous
calcium gluconate.

Management of hyperkalaemia
Untreated hyperkalaemia may cause life-threatening arrhythmias. Precipitating factors
should be addressed (e.g. acute renal failure) and aggravating drugs stopped (e.g.
ACE inhibitors). Management may be categorised by the aims of treatment

Stabilisation of the cardiac membrane

 Intravenous calcium gluconate

Short-term shift in potassium from extracellular to intracellular fluid compartment

 Combined insulin/dextrose infusion


 Nebulised salbutamol

Removal of potassium from the body

 Calcium resonium (orally or enema)


 Loop diuretics
 Dialysis

Theme: Visceral injury

Ruptured spleen A.
Ileum injury B.
Duodenal injury C.
Urethral injury D.
Rectal injury E.
Oesophageal injury F.
Liver laceration G.

Please select the most likely injury for the scenario given. Each option may be used
once, more than once or not at all.

A motorcyclist is involved in a head on road traffic accident with a lorry. He is 38.


comatose at scene and trauma series xrays confirm a pelvic fracture. On rectal
examination he has a high riding prostate.

Urethral injury

This is classical for urethral injury. Features of a urethral injury include; pelvic
fracture, high riding prostate on digital rectal examination and blood at the
urethral meatus. Where this is the suspected diagnosis a suprapubic catheter
and urethral contrast studies performed.
A cyclist loses control and falls off the side of a road landing on the bicycle 39.
handlebars. CT scanning shows a large amount of retroperitoneal air.

Duodenal injury

Retroperitoneal air is more likely with a duodenal injury. As it is largely


retroperitoneal. A handlebar type injury is the commonest cause and the
pancreas should be carefully inspected as it too may be injured. It would be
unusual for the ileum to be injured in this type of scenario as it is mobile.

A 23 year old man is shot in the abdomen. He is haemodynamically stable but 40.
on ultrasound he has a large about of intra abdominal free fluid.

Ileum injury

Small bowel injury is the most common type of injury in this scenario. The
enteric contents will tend to result in a large amount of intra abdominal fluid.

Trauma management

The cornerstone of trauma management is embodied in the principles of ATLS.

Following trauma there is a trimodal death distribution:

 Immediately following injury. Typically as result of brain or high spinal


injuries, cardiac or great vessel damage. Salvage rate is low.
 In early hours following injury. In this group deaths are due to phenomena
such as splenic rupture, sub dural haematomas and haemopneumothoraces
 In the days following injury. Usually due to sepsis or multi organ failure.

Aspects of trauma management

 ABCDE approach.
 Tension pneumothoraces will deteriorate with vigorous ventilation attempts.
 External haemorrhage is managed as part of the primary survey. As a rule
tourniquets should not be used. Blind application of clamps will tend to
damage surrounding structures and packing is the preferred method of
haemorrhage control.
 Urinary catheters and naso gastric tubes may need inserting. Be wary of basal
skull fractures and urethral injuries.
 Patients with head and neck trauma should be assumed to have a cervical
spine injury until proven otherwise.
Thoracic injuries

 Simple pneumothorax
 Mediastinal traversing wounds
 Tracheobronchial tree injury
 Haemothorax
 Blunt cardiac injury
 Diaphragmatic injury
 Aortic disruption
 Pulmonary contusion

Management of thoracic trauma

 Simple pneumothorax insert chest drain. Aspiration is risky in trauma as


pneumothorax may be from lung laceration and convert to tension
pneumothorax.
 Mediastinal traversing wounds These result from situations like stabbings.
Exit and entry wounds in separate hemithoraces. The presence of a mediastinal
haematoma indicates the likelihood of a great vessel injury. All patients should
undergo CT angiogram and oesophageal contrast swallow. Indications for
thoracotomy are largely related to blood loss and will be addressed below.
 Tracheobronchial tree injury Unusual injuries. In blunt trauma most injuries
occur within 4cm of the carina. Features suggesting this injury include
haemoptysis and surgical emphysema. These injuries have a very large air leak
and may have tension pneumothorax.
 Haemothorax Usually caused by laceration of lung vessel or internal
mammary artery by rib fracture. Patients should all have a wide bore 36F chest
drain. Indications for thoracotomy include loss of more than 1.5L blood
initially or ongoing losses of >200ml per hour for >2 hours.
 Cardiac contusions Usually cardiac arrhythmias, often overlying sternal
fracture. Perform echocardiography to exclude pericardial effusions and
tamponade. Risk of arrhythmias falls after 24 hours.
 Diaphragmatic injury Usually left sided. Direct surgical repair is performed.
 Traumatic aortic disruption Commonest cause of death after RTA or falls.
Usually incomplete laceration near ligamentum arteriosum. All survivors will
have contained haematoma. Only 1-2% of patients with this injury will have a
normal chest x-ray.
 Pulmonary contusion Common and lethal. Insidious onset. Early intubation
and ventilation.

Abdominal trauma

 Deceleration injuries are common.


 In blunt trauma requiring laparotomy the spleen is most commonly injured
(40%)
 Stab wounds traverse structures most commonly liver (40%)
 Gunshot wounds have variable effects depending upon bullet type. Small
bowel is most commonly injured (50%)
 Patients with stab wounds and no peritoneal signs up to 25% will not enter the
peritoneal cavity
 Blood at urethral meatus suggests a urethral tear
 High riding prostate on PR = urethral disruption
 Mechanical testing for pelvic stability should only be performed once

Investigations in abdominal trauma

USS Abdominal CT scan Diagnostic


Peritoneal Lavage
Document fluid if Document organ injury Document bleeding if Indication
hypotensive if normotensive hypotensive
Early diagnosis, non Most specific for Early diagnosis and Advantages
invasive and localising injury; 92 to sensitive; 98%
repeatable; 86 to 95% 98% accurate accurate
accurate
Operator dependent Location of scanner Invasive and may Disadvantages
and may miss away from facilities, miss retroperitoneal
retroperitoneal injury time taken for reporting, and diaphragmatic
need for contrast injury

 Amylase may be normal following pancreatic trauma


 Urethrography if suspected urethral injury

A 22 year old man has a full thickness burn of his leg after being trapped in a burning
car. There are no fractures of the limb. There burn is well circumscribed. After 2
hours he complains of tingling of his leg and it appears dusky. What is the best
management for this?

Fasciotomy A.

Escharotomy B.

Angioplasty C.

Pain control D.

Anticoagulation E.

The full thickness burn has oedema which is affecting the peripheral circulation.
Therefore the burn needs to be divided (not the fascia) to allow normal circulation to
return.

Burns
Types of burn

Management Blanching Skin Skin layers Type of burn


appearance affected
Yes Red, moist Epidermis Epidermal/Superficial
Normally heals Yes Pale, dry Epidermis and Superficial partial
with no part of papillary thickness
intervention dermis affected
Needs surgical No Mottled red Epidermis, whole Deep partial thickness
intervention colour papillary dermis
(depending on affected
site)
Burns centre No Dry, leathery Whole skin layer Full thickness
hard wound and subcutaneous
tissue affected

Depth of burn assessment

 Bleeding on needle prick


 Sensation
 Appearance
 Blanching to pressure

Percentage burn estimation


Lund Browder chart: most accurate even in children
Wallace rule of nines
Palmar surface: surface area palm = 0.8% burn

>15% body surface area burns in adults needs urgent burn fluid resuscitation

Transfer to burn centre if:

 Need burn shock resuscitation


 Face/hands/genitals affected
 Deep partial thickness or full thickness burns
 Significant electrical/chemical burns

Escharotomies

 Indicated in circumferential full thickness burns to the torso or limbs.


 Careful division of the encasing band of burn tissue will potentially improve
ventilation (if the burn involves the torso), or relieve compartment syndrome
and oedema (where a limb is involved)

References
www.euroburn.org/e107files/downloads/guidelinesburncare.pdf
Hettiaratchy S & Papini R. Initial management of a major burn: assessment and
resuscitation. BMJ 2004;329:101-103
A 28 year old man is in the surgical intensive care unit. He has suffered a flail chest
injury several hours earlier and he was intubated and ventilated. Over the past few
minutes he has become increasingly hypoxic and is now needing increased ventilation
pressures. What is the most common cause?

Pulmonary embolism A.

Cardiac tamponade B.

Fat embolism C.

Tension pneumothorax D.

Adult respiratory distress syndrome E.

Theme from April 2011 Exam

A flail chest segment may lacerate the underlying lung and create a flap valve. A
tension pneumothorax can be created by intubation and ventilation in this situation.
Sudden hypoxia and increased ventilation pressure are clues.

Tension Pneumothorax

 May occur following thoracic trauma when a lung parenchymal flap is created.
 This acts as a one way valve and allows pressure to rise.
 The trachea shifts and hyper-resonance is apparent on the affected side.
 Treatment is with needle cricothyroidotomy and chest tube insertion.

Image showing mediastinal shift with a tension pneumothorax


Image sourced from Wikipedia
A 10 year old boy is playing with a firework which explodes and he sustains a full
thickness burn to his left arm. Which of the following statements is not characteristic
of this situation?

They have a leathery appearance A.

The burn area is extremely painful until skin grafted B.

They always heal with scarring C.

Blanching does not occur under pressure D.

Absence of,or few, blisters E.

Full thickness burns involve complete injury to the dermis and sub dermal
appendages. They have a leathery, often white appearance. They are initially insensate
although pain often occurs during healing following skin grafting. They do not blanch
under pressure.

Burns

Types of burn

Management Blanching Skin Skin layers Type of burn


appearance affected
Yes Red, moist Epidermis Epidermal/Superficial
Normally heals Yes Pale, dry Epidermis and Superficial partial
with no part of papillary thickness
intervention dermis affected
Needs surgical No Mottled red Epidermis, whole Deep partial thickness
intervention colour papillary dermis
(depending on affected
site)
Burns centre No Dry, leathery Whole skin layer Full thickness
hard wound and subcutaneous
tissue affected

Depth of burn assessment

 Bleeding on needle prick


 Sensation
 Appearance
 Blanching to pressure

Percentage burn estimation


Lund Browder chart: most accurate even in children
Wallace rule of nines
Palmar surface: surface area palm = 0.8% burn

>15% body surface area burns in adults needs urgent burn fluid resuscitation

Transfer to burn centre if:

 Need burn shock resuscitation


 Face/hands/genitals affected
 Deep partial thickness or full thickness burns
 Significant electrical/chemical burns

Escharotomies

 Indicated in circumferential full thickness burns to the torso or limbs.


 Careful division of the encasing band of burn tissue will potentially improve
ventilation (if the burn involves the torso), or relieve compartment syndrome
and oedema (where a limb is involved)

References
www.euroburn.org/e107files/downloads/guidelinesburncare.pdf

Hettiaratchy S & Papini R. Initial management of a major burn: assessment and


resuscitation. BMJ 2004;329:101-103
Theme: Thoracic injuries

Pneumothorax A.
Tension pneumothorax B.
Flail chest C.
Cardiac tamponade D.
Aorta rupture E.
Cardiac contusion F.
Diaphragmatic rupture G.
Acute phrenic nerve injury H.

For each of the scenarios given, please select the most likely underlying injury. Each
option may be used once, more than once or not at all.

An 18 year old student is involved in a car crash, with another car crashing 44.
into the side of the car.
A CXR shows an indistinct left hemidiaphragm.

You answered Acute phrenic nerve injury

The correct answer is Diaphragmatic rupture

A lateral blunt injury during a road traffic accident is a common cause of


diaphragmatic rupture. Diagnosis is usually evident on chest x-ray. CXR
changes include non visible diaphragm, bowel loops in the hemithorax and
displacement of the mediastinum. In most cases direct surgical repair is the
best option.

A 19 year old motorcyclist is involved in a road traffic accident. His chest 45.
movements are irregular. He is found to have multiple rib fractures, with 2
fractures in the 3rd rib and 3 fractures in the 4th rib.

Flail chest

Multiple rib fractures with > or = 2 rib fractures in more than 2 ribs is
diagnosed as a flail chest. This is associated with pulmonary contusion.

A 19 year old student falls from a 2nd floor window. He is persistently 46.
hypotensive. A CXR shows depression of the left main bronchus and deviation
of the trachea to the right.

Aorta rupture

He has a deceleration injury, with persistent hypotension (contained


haematoma). This should indicate aorta rupture. Widened mediastinum may
not always be present on a CXR. A CT angiogram will provide clearer
evidence of the extent of injury. The presence of persistent hypotension, from
a early stage is more consistent with haematoma than a tension pneumothorax
in which it occurs as a final periarrest phenomena.

CXR findings in diaphragmatic rupture:


Hemidiaphragm is not visible
Bowel loops in the lower half of the
hemi-thorax
Mediastinum is displaced

Thoracic trauma

Types of thoracic trauma

 Often laceration to lung parenchyma with flap Tension


 Pressure develops in thorax pneumothorax
 Most common cause is mechanical ventilation in patient
with pleural injury
 Symptoms overlap with cardiac tamponade, hyper-
resonant percussion note is more likely in tension
pnemothorax

 Chest wall disconnects from thoracic cage Flail chest


 Multiple rib fractures (at least two fractures per rib in at
least two ribs)
 Associated with pulmonary contusion
 Abnormal chest motion
 Avoid over hydration and fluid overload

 Most common cause is lung laceration with air leakage Pneumothorax


 Most traumatic pneumothoraces should have a chest
drain
 Patients with traumatic pneumothorax should never be
mechanically ventilated until a chest drain is inserted

 Most commonly due to laceration of lung, intercostal Haemothorax


vessel or internal mammary artery
 Haemothoraces large enough to appear on CXR are
treated with large bore chest drain
 Surgical exploration is warranted if >1500ml blood
drained immediately

 Beck's triad: elevated venous pressure, reduced arterial Cardiac tamponade


pressure, reduced heart heart sounds.
 Pulsus paradoxus
 May occur with as little as 100ml blood
 Most common potentially lethal chest injury Pulmonary
 Arterial blood gases and pulse oximetry important contusion
 Early intubation within an hour if significant hypoxia

 Usually occurs secondary to chest wall injury Blunt cardiac injury


 ECG may show features of myocardial infarction
 Sequelae: hypotension, arrhythmias, cardiac wall motion
abnormalities

 Deceleration injuries Aorta disruption


 Contained haematoma
 Widened mediastinum

 Most due to motor vehicle accidents and blunt trauma Diaphragm


causing large radial tears (laceration injuries result in disruption
small tears)
 More common on left side
 Insert gastric tube, which will pass into the thoracic
cavity

 Entrance wound in one haemothorax and exit Mediastinal


wound/foreign body in opposite haemothorax traversing wounds
 Mediastinal haematoma or pleural cap suggests great
vessel injury
 Mortality is 20%

References
ATLS Manual 8th Edition
A 22 year old man is involved in a motorcycle accident. He suffers from bilateral
tibial and fibula fractures. He is taken to theatre and intramedullary nails are inserted.
6h after surgery he complains of severe pain in his limb, which increases on passive
plantar flexion. What is the best management plan?

Anticoagulation A.

Fasciotomy B.

Intravenous analgesia and clinical review in 8 C.


hours

Exploration of tibia for displaced nail D.

Pregabalin E.
Do not forget to decompress the deep
muscle layer during a fasciotomy.

This patient has compartment syndrome and needs a fasciotomy.


Compartment syndrome

 This is a particular complication that may occur following fractures (or


following ischaemia reperfusion injury in vascular patients). It is characterised
by raised pressure within a closed anatomical space.
 The raised pressure within the compartment will eventually compromise tissue
perfusion resulting in necrosis. The two main fractures carrying this
complication include supracondylar fractures and tibial shaft injuries.

Symptoms and signs

 Pain, especially on movement (even passive)


 Parasthesiae
 Pallor may be present
 Arterial pulsation may still be felt as the necrosis occurs as a result of
microvascular compromise
 Paralysis of the muscle group may occur

Diagnosis

 Is made by measurement of intracompartmental pressure measurements.


Pressures in excess of 20mmHg are abnormal and >40mmHg is diagnostic.

Treatment

 This is essentially prompt and extensive fasciotomies


 In the lower limb the deep muscles may be inadequately decompressed by the
inexperienced operator when smaller incisions are performed
 Myoglobinuria may occur following fasciotomy and result in renal failure and
for this reason these patients require aggressive IV fluids
 Where muscle groups are frankly necrotic at fasciotomy they should be
debrided and amputation may have to be considered
 Death of muscle groups may occur within 4-6 hours
 Which of the features below, following a head injury, is not an indication for
an immediate CT head scan in children?

Drowsiness A.

A single, discrete episode of vomiting B.

A 9 month old child with a 6cm haematoma on the head C.

Numb left arm D.


Suspicion of a non accidental head injury E.

Whilst not an indication for immediate CT there should be a low threshold for
admission and observation.
 Head injury paediatrics

Criteria for immediate request for CT scan of the head (children)

* Loss of consciousness lasting more than 5 minutes (witnessed)


* Amnesia (antegrade or retrograde) lasting more than 5 minutes
* Abnormal drowsiness
* Three or more discrete episodes of vomiting
* Clinical suspicion of non-accidental injury
* Post-traumatic seizure but no history of epilepsy
* GCS less than 14, or for a baby under 1 year GCS (paediatric) less than 15,
on assessment in the emergency department
* Suspicion of open or depressed skull injury or tense fontanelle
* Any sign of basal skull fracture (haemotympanum, ‘panda' eyes,
cerebrospinal fluid leakage from the ear or nose, Battle's sign)
* Focal neurological deficit
* If under 1 year, presence of bruise, swelling or laceration of more than 5 cm
on the head
* Dangerous mechanism of injury (high-speed road traffic accident either as
pedestrian, cyclist or vehicle occupant, fall from a height of greater than 3 m,
high-speed injury from a projectile or an object)
Theme: Management of osteomyelitis

Lautenbach regime A.
Below knee amputation B.
Hindquater amputation C.
Above knee amputation D.
Removal of metalwork and implantation of local antibiotics E.
Removal of metalwork and bone grafting F.
Intravenous antibiotics G.

Which option is the best management plan? Each option may be used once, more than
once or not at all

A 65 year old type 2 diabetic with poor glycaemic control is admitted with 3.
forefoot cellulitis. X-ray of the foot shows some evidence of osteomyelitis of
the 2nd ray but overlying skin is healthy.

Intravenous antibiotics

It is worth attempting to try and resolve this situation with antibiotics at first
presentation. A primary amputation will not heal well and may result in
progressive surgery.

A 28 year old infantryman is shot in the leg during combat. Primary 4.


debridement and lavage of the wound is undertaken. Several months post
surgery there is ongoing discharge from a sinus originating in the proximal
femur, X-ray and MRI shows evidence of osteomyelitis of the proximal femur.
There are no obvious sequestra.

You answered Removal of metalwork and bone grafting

The correct answer is Lautenbach regime

This involves local administration of antibiotics via intramedullary lines and is


an intensive regime. However, the morbidity of a high above knee or hindquater
amputation makes conservative management an attractive option.

A 70 year old man undergoes a revision total hip replacement. 10 days post 5.
operatively the hip dislocates and pus is discharging from the wound. He is
systemically unwell with a temperature of 38.5 and WCC 19.

Removal of metalwork and implantation of local antibiotics

Removal of metal work implantation of gentamicin beads and delayed revision


is the mainstay of managing this complication.

Osteomyelitis

Infection of the bone

Causes

 S aureus and occasionally Enterobacter or Streptococcus species


 In sickle cell: Salmonella species

Clinical features

 Erythema
 Pain
 Fever

Investigation
 X-ray: lytic centre with a ring of sclerosis
 Bone biopsy and culture

Treatment

 Prolonged antibiotics
 Sequestra may need surgical removal

What is the least likely examination finding in patients with Le Fort II fractures?

Excessive mobility of the palate A.

Paraesthesia in the region supplied by the inferior alveolar nerve B.

Malocclusion of the teeth C.

Endopthalmos D.

Parasthesia in the region supplied by the infraorbital nerve E.

Le Fort II fractures have a pyramidal shape. The fracture line involves the orbit and
extends to involve the bridge of the nose and the ethmoids. In continues to involve the
infraorbital rim and usually through the infraorbital foramen. As a result infraorbital
parasthesia, palatal mobility and malocclusion are common findings. Severe fractures
may result in endopthalmos. However, the fracture does not, by definition, involve the
inferior alveolar nerve.

Craniomaxillofacial injuries

Craniomaxillofacial injuries in the UK are due to:

 Interpersonal violence (52%)


 Motor vehicle accidents (16%)
 Sporting injuries (19%)
 Falls (11%)

Le Fort Fractures
Feature Grade
The fracture extends from the nasal septum to the lateral pyriform rims, travels Le Fort
horizontally above the teeth apices, crosses below the zygomaticomaxillary 1
junction, and traverses the pterygomaxillary junction to interrupt the pterygoid
plates.
These fractures have a pyramidal shape and extend from the nasal bridge at or Le Fort
below the nasofrontal suture through the frontal process of the maxilla, 2
inferolaterally through the lacrimal bones and inferior orbital floor and rim
through or near the inferior orbital foramen, and inferiorly through the anterior
wall of the maxillary sinus; it then travels under the zygoma, across the
pterygomaxillary fissure, and through the pterygoid plates.
These fractures start at the nasofrontal and frontomaxillary sutures and extend Le Fort
posteriorly along the medial wall of the orbit through the nasolacrimal groove 3
and ethmoid bones. The thicker sphenoid bone posteriorly usually prevents
continuation of the fracture into the optic canal. Instead, the fracture continues
along the floor of the orbit along the inferior orbital fissure and continues
superolaterally through the lateral orbital wall, through the zygomaticofrontal
junction and the zygomatic arch. Intranasally, a branch of the fracture extends
through the base of the perpendicular plate of the ethmoid, through the vomer,
and through the interface of the pterygoid plates to the base of the sphenoid.
This type of fracture predisposes the patient to CSF rhinorrhea more
commonly than the other types.

Ocular injuries
Superior orbital fissure syndrome
Severe force to the lateral wall of the orbit resulting in compression of neurovascular
structures. Results in :

 Complete opthalmoplegia and ptosis (Cranial nerves 3, 4, 6 and nerve to


levator palpebrae superioris)
 Relative afferent pupillary defect
 Dilatation of the pupil and loss of accommodation and corneal reflexes
 Altered sensation from forehead to vertex (frontal branch of trigeminal nerve)

Orbital blow out fracture


Typically occurs when an object of slightly larger diameter than the orbital rim strikes
the incompressible eyeball. The bone fragment is displaced downwards into the antral
cavity, remaining attached to the orbital periosteum. Periorbital fat may be herniated
through the defect, interfering with the inferior rectus and inferior oblique muscles
which are contained within the same fascial sheath. This prevents upward movement
and outward rotation of the eye and the patient experiences diplopia on upward gaze.
The initial bruising and swelling may make assessment difficult and patients should
usually be reviewed 5 days later. Residual defects may require orbital floor
reconstruction.

Nasal Fractures

 Common injury
 Ensure new and not old deformity
 Control epistaxis
 CSF rhinorrhoea implies that the cribriform plate has been breached and
antibiotics will be required.
 Usually best to allow bruising and swelling to settle and then review patient
clinically. Major persistent deformity requires fracture manipulation, best
performed within 10 days of injury.
Retrobulbar haemorrhage
Rare but important ocular emergency. Presents with:

 Pain (usually sharp and within the globe)


 Proptosis
 Pupil reactions are lost
 Paralysis (eye movements lost)
 Visual acuity is lost (colour vision is lost first)

May be the result of Le Fort type facial fractures.

Management:

 Mannitol 1g/Kg as 20% infusion, Osmotic diuretic, Contra-indicated in


congestive heart failure and pulmonary oedema
 Acetazolamide 500mg IV, (Monitor FBC/U+E) Reduces aqueous pressure by
inhibition of carbonic anhydrase (used in glaucoma)
 Dexamethasone 8mg orally or intravenously
 In a traumatic setting an urgent catholysis may be needed prior to definitive
surgery.

Consider
Papaverine 40mg smooth muscle relaxant
Dextran 40 500mls IV improves perfusion
A Medical F1 phones you as he is concerned his patient has had a major internal
bleed. The patient is 42 years old and is known to have sickle cell anaemia. His blood
results are:

Hb 3.7 g /dl
Reticulocyte count 0.4%

His Hb is normally 7g/dl. What is the diagnosis?

Psoas haemorrhage A.

Acute sequestration B.

Parvovirus C.

Splenic haemorrhage D.

Acute haemolysis E.

A sudden anemia and a LOW reticulocute count indicates parvovirus. Acute


sequestration and haemolysis causes a high reticulocyte count. There is no clinical
indication to suspect a bleed, therefore you can advise the F1 not to panic and to
speak to the haematologists!

Sickle cell anaemia

 Autosomal recessive
 Single base mutation
 Deoxygenated cells become sickle in shape
 Causes: short red cell survival, obstruction of microvessels and infarction
 Sickling is precipitated by: dehydration, infection, hypoxia
 Manifest at 6 months age
 Africans, Middle East, Indian
 Diagnosis: Hb electrophoresis

Sickle crises

 Bone pain
 Pleuritic chest pain: acute sickle chest syndrome commonest cause of death
 CVA, seizures
 Papillary necrosis
 Splenic infarcts
 Priapism
 Hepatic pain

Hb does not fall during a crisis, unless there is

 Aplasia: parvovirus
 Acute sequestration
 Haemolysis

Long-term complications

 Infections: Streptococcus pnemoniae


 Chronic leg ulcers
 Gallstones: haemolysis
 Aseptic necrosis of bone
 Chronic renal disease
 Retinal detachment, proliferative retinopathy

Surgical complications

 Bowel ischaemia
 Cholecystitis
 Avascular necrosis
Management

 Supportive
 Hydroxyurea
 Repeated transfusions pre operatively
 Exchange transfusion in emergencies

Sickle cell trait

 Heterozygous state
 Asymptomatic
 Symptoms associated with extreme situations ie anaesthesia complications
 Protective against Plasmodium falciparum

Theme: Management of chest trauma

Thoracotomy in operating theatre A.


36F intercostal chest drain B.
14F intercostal chest drain C.
Active observation D.
Thoracotomy in the emergency room E.
MRI of aortic arch F.
Bronchoscopy G.
Pericardiocentesis H.
Further transfusion I.

For each of the following scenarios please select the most appropriate next stage of
management. Each option may be used once, more than once or not at all.

A 30 year old male is stabbed outside a nightclub he has a brisk haemoptysis 8.


and in casualty has a chest drain inserted into the left chest. This drained
750ml frank blood. He fails to improve with this intervention. He has received
4 units of blood. His CVP is now 13.

You answered Thoracotomy in operating theatre

The correct answer is Pericardiocentesis

This man has cardiac tamponade. The raised CVP in the setting of
haemodynamic compromise is the pointer to this. Whilst he will almost
cetainly require surgery, he requires ungent deompresion of his heart first.

A 26 year old male falls from a cliff. He suffers from multiple fractures and 9.
has a right sided pneumothorax that has collapsed a 1/3 of his lung. He has no
respiratory compromise.

14F intercostal chest drain

Simple observation is unsafe as he will almost certainly have suffered an


oblique laceration to his lung. These can become tension pneumothoraces. In
the absence of blood a 36 F drain is probably not required

An 18 year old male is shot in the left chest he was unstable but his blood 10.
pressure has improved with 1 litre of colloid. His chest x-ray shows a left sided
pneumothorax with no lung visible.

36F intercostal chest drain

This man requires wide bore intercostal tube drainage. Smaller intercostal
chest drains can become occluded with blood clot and fail to function
adaquetly.

Thoracic trauma

Types of thoracic trauma

 Often laceration to lung parenchyma with flap Tension


 Pressure develops in thorax pneumothorax
 Most common cause is mechanical ventilation in patient
with pleural injury
 Symptoms overlap with cardiac tamponade, hyper-
resonant percussion note is more likely in tension
pnemothorax

 Chest wall disconnects from thoracic cage Flail chest


 Multiple rib fractures (at least two fractures per rib in at
least two ribs)
 Associated with pulmonary contusion
 Abnormal chest motion
 Avoid over hydration and fluid overload

 Most common cause is lung laceration with air leakage Pneumothorax


 Most traumatic pneumothoraces should have a chest
drain
 Patients with traumatic pneumothorax should never be
mechanically ventilated until a chest drain is inserted

 Most commonly due to laceration of lung, intercostal Haemothorax


vessel or internal mammary artery
 Haemothoraces large enough to appear on CXR are
treated with large bore chest drain
 Surgical exploration is warranted if >1500ml blood
drained immediately

 Beck's triad: elevated venous pressure, reduced arterial Cardiac tamponade


pressure, reduced heart heart sounds.
 Pulsus paradoxus
 May occur with as little as 100ml blood

 Most common potentially lethal chest injury Pulmonary


 Arterial blood gases and pulse oximetry important contusion
 Early intubation within an hour if significant hypoxia

 Usually occurs secondary to chest wall injury Blunt cardiac injury


 ECG may show features of myocardial infarction
 Sequelae: hypotension, arrhythmias, cardiac wall motion
abnormalities

 Deceleration injuries Aorta disruption


 Contained haematoma
 Widened mediastinum

 Most due to motor vehicle accidents and blunt trauma Diaphragm


causing large radial tears (laceration injuries result in disruption
small tears)
 More common on left side
 Insert gastric tube, which will pass into the thoracic
cavity

 Entrance wound in one haemothorax and exit Mediastinal


wound/foreign body in opposite haemothorax traversing wounds
 Mediastinal haematoma or pleural cap suggests great
vessel injury
 Mortality is 20%

References
ATLS Manual 8th Edition
Theme: Management of burns

Escharotomy A.
Endotracheal intubation B.
Broad spectrum intravenous antibiotics C.
Intravenous fluids calculated according to extent of burned area D.
Discharge with review in outpatients E.
Transfer to regional burn centre once stabilised F.
Split thickness skin graft G.
Full thickness skin graft H.

What is the best management for the scenario given? Each option may be used once,
more than once or not at all.

A 34 year old women trips over and falls into a bonfire whilst intoxicated at a 11.
party. She suffers burns to her arms, torso and face. These are calculated to be
25% body surface area. She is otherwise stable. The burns to the torso are
superficial, her left forearm has a full thickness burn and the burns to her face
are superficial. There is no airway compromise. She has received 1000ml of
intravenous Hartman's solution, with a further 1000ml prescribed to run over 4
hours.

Transfer to regional burn centre once stabilised

This women has been resuscitated and requires transfer for specialist
management

A 20 year old man is trapped in a warehouse fire. He has sustained 60% burns 12.
to his torso and limbs. The limb burns are partial thickness but the torso burns
are full thickness. He was intubated by paramedics at the scene and is
receiving intravenous fluids. His ventilation pressure requirements are rising.

Escharotomy

He requires an escharotomy as this will be contributing to impaired ventilation

An 18 year old man accidentally pours boiling water onto his left arm. The 13.
area is erythematous and has a blister measuring 5cm. The wound is extremely
painful.

Discharge with review in outpatients

This is a superficial burn and should recover with no further input than simple
dressings, an alternative would be deroofing the blister and applying dressings
prior to outpatient review

Burns

Types of burn

Management Blanching Skin Skin layers Type of burn


appearance affected
Yes Red, moist Epidermis Epidermal/Superficial
Normally heals Yes Pale, dry Epidermis and Superficial partial
with no part of papillary thickness
intervention dermis affected
Needs surgical No Mottled red Epidermis, whole Deep partial thickness
intervention colour papillary dermis
(depending on affected
site)
Burns centre No Dry, leathery Whole skin layer Full thickness
hard wound and subcutaneous
tissue affected

Depth of burn assessment

 Bleeding on needle prick


 Sensation
 Appearance
 Blanching to pressure

Percentage burn estimation


Lund Browder chart: most accurate even in children
Wallace rule of nines
Palmar surface: surface area palm = 0.8% burn

>15% body surface area burns in adults needs urgent burn fluid resuscitation

Transfer to burn centre if:

 Need burn shock resuscitation


 Face/hands/genitals affected
 Deep partial thickness or full thickness burns
 Significant electrical/chemical burns

Escharotomies

 Indicated in circumferential full thickness burns to the torso or limbs.


 Careful division of the encasing band of burn tissue will potentially improve
ventilation (if the burn involves the torso), or relieve compartment syndrome
and oedema (where a limb is involved)

References
www.euroburn.org/e107files/downloads/guidelinesburncare.pdf

Hettiaratchy S & Papini R. Initial management of a major burn: assessment and


resuscitation. BMJ 2004;329:101-103
A 56-year-old female is admitted to ITU with a severe pancreatitis. Thyroid function
tests show:
TSH = 0.5 Low
Thyroxine = 1.0 Low
T3 = 0.5 Low

What is the most likely cause?

Sick euthyroid syndrome A.

Graves disease B.

Hashimotos thyroiditis C.

Levothyroxine D.

None of the above E.

This patient has sick euthyroid syndrome as all thyroid parameters are reduced.
Graves disease and levothyroxine will cause hyperthyroidism (low TSH and elevated
thyroxine/T3). Hashimotos thyroiditis is associated with hypothyroidism (high TSH
and low thyroxine/T3).

Sick euthyroid syndrome

In sick euthyroid syndrome (now referred to as non-thyroidal illness) it is often said


that everything (TSH, thyroxine and T3) is low. In the majority of cases however the
TSH level is within the normal range (inappropriately normal given the low thyroxine
and T3).

Changes are reversible upon recovery from the systemic illness.

You might also like